SlideShare ist ein Scribd-Unternehmen logo
1 von 172
Male sexual dysfunction
Edmond Wong
A
B
C
• Name the structures A, B, C and D (0.5 mark each)
D
• A : Skin (0.5)
• B : Dartos fascia (0.5)
• C : Buck’s fascia (0.5)
• D : Tunica albuginea (0.5)
What is the definition of ED?
• Persistent inability to initiate and maintain
an erection sufficient for satisfactory
sexual activity
• Better not to use “Impotence” as it is less
precisely defined
What is prevalence & severity?
• Massachusetts Male Aging Study (MMAS)
• Prevalence –
– Men 40-70yo
• ~50% have ED
• Mild ED: 17%
• Moderate ED : 25%
• Complete ED : 10%
• ED prevalence increases with age
– 50% at age 50, 60% at age 60, 70% at age 70
What is innervation of
erection and ejaculation?
• Autonomic
– Sympathetic nerves from T11-L2
– Parasympathetic from S2-4, form the pelvic plexus
– The cavernosal nerves are branches of pelvic plexus
(i.e. parasympathetic) that innervate the penis
– Parasympathetic stimulation causes erection
– Sympathetic activity causes ejaculation and
detumescence (loss of erection)
What is innervation of
erection and ejaculation?
• Somatic
– Somatosensory information travels via the pudendal
nerves
– Onuf’s nucleus (S2-4) is the somatic centre for
efferent innervation of the ischiocavernosus and
bulbocavernosus muscles of the penis
• Central
– Medial preoptic area and paraventricular nucleus
(PVN) in the hypothalamus are important centres for
sexual function and penile erection
What is the arterial blood
supply of penis?
• Originate from internal pudendal artery
• Three branches of common penile artery which join to form
vascular ring near the glans
• Bulbourethral artery
– Supplying the bulb and corpus spongiosum
• Dorsa penile artery
– Skin, fascia and the glans penis and forms anastomoses with the
bulbourethral artery at he glans. These anastomoses allow division the
the urethra during urethral stricture surgery without compromising the
blood supply to the distal urethral
• The cavernous artery
– Supplies only the cavernosal bodies & gives off many helicine arteries
which supply the trabecular erectile tissue & sinusoid. Does not
anastomose with other 2 penile arteries
• Pulsations absent in penile vessels – likely vascular cause of
ED
What is the venous blood
supply of penis?
• Skin and subcutaneous tissue  saphenous
vein
• Emissary veins draining corpus cavernosum and
corpus spongiosum  deep dorsal vein 
periprostatic venous plexus
• Emissary veins draining proximal corpora
cavernosa  nternal pudendal veins
Physiology of Erectile ResponsePhysiology of Erectile Response
Complex process
combining
• psychological stimuli
• neurologic event
• smooth muscle relaxation
• arterial dilation
• venous compression
What are the five phases of
erection?
Phase Term
0 Flaccid phase
1 Latent (filling) phase
2 Tumescent phase
3 Full erection phase
4 Rigid erection phase
5 Detumescence phase
What is mechanism of
erection?
• Neuroendocrine signals from the brain, created by audiovisual
or tactile stimuli
• Signals are relayed via the cavernosal nerve to the erectile
tissue of the copora cavernosa, activating the veno-occlusive
mechanism
• This triggers increased arterial blood flow into sinusoidal spaces
with relaxation of cavernosal smooth muscle, and opening of
the vascular space
• Compressing the subtunical venous plexuses, decreasing
venous outflow
• Both spongiosus and cavernosus are surrounded by tunica
albuginea, which consist of outer longitudinal and inner circular
layers. The sliding of 2 layers over each other during
engorgement lead to occlusion of emissary veins
• Rising intracavernosal pressure and contraction of the
ischiocavernosus muscles produces a rigid erection
Mechanism of erection
• Full erection phase:
– Compression of the deep dorsal and
cricumflex vein btw Buck’s fascia & engorged
cavernosa  glanular tumescence
• Rigid –erection phase:
– Ischiocavernosus and bulbocavernosus
muscle forcefully compress the spongiosum
and penile veins  further engorgement and
increase pressure in glans and spongiosum
Mechanism of PDE5-i
• nitric oxide is synthesized from L-arginine and released
by neurons, endothelial cells, and possibly corporal
smooth muscle cells of the penis in response to sexual
stimulation
• Nitric oxide enter SM cell
• Activate soluble form of enzyme guanylate cyclase
(sGC)
• sGC convert guanosine triphosphate (GTP) to cyclic
guanosine monophosphate (cGMP) (an active
intracellular 2nd
messenger)
• cGMP lead to SM relaxation thru reduction of
intracellular Ca  erection
• cGMP is metabolize to GMP (inactive) by PDEs
• Thus PDE inhibitor facilitate NO induced SM relaxation
by increase accumulation of intracellular cGMP
What is mechanism of
erection?
Pathophysiology of Erectile Dysfunction
Inflow Outflow
Failure to initiate
Psychogenic / Neurogenic
Failure to store
Venous leak
Failure to fill
Arterial insufficiency
What is the mechanism of
venogenic erectile dysfunction?
• What physiological process is this chart describing? (1)
• What are the top and bottom row depicting? (2)
• Can you name the 6 columns? (2)
Q57
• Tumescence / erection (1)
• Top row : artist depiction of the state of the
cavernosal arteries during different phases of
erection induced by prostaglandin injection (1)
• Bottom row : Doppler waveform of the
cavernous arteries during this erection (1)
• Flaccid, latent, tumescent, full, rigid,
detumescent
• (2 marks for ALL 6 correct answers)
What is the mechanism of
ejaculation?
• Tactile stimulation of the glans penis causes sensory
information to travel (via the pudendal nerve) to the
lumbar spinal sympathetic nuclei
• Sympathetic efferent signals (travelling in the
hypogastric nerve) cause contraction of smooth muscle
of the epididymis, vas deferens, and secretory glands,
propelling spermatozoa and glandular secretions into the
prostatic urethra
• There is simultaneous closure of the internal urethral
sphincter and relaxation of the extrinsic sphincter
• Rhythmic contraction of the bulbocavernosus muscle
leads to the pulsatile emission of the ejaculate from the
urethra
Risk Factors for ED
• Aging
• Systemic diseases: D.M., hypertension,
atherosclerosis, hyperlipidemia
• Endocrine disorders: hypogonadism,
hyperprotactinemia
• Alcohol abuse or smoking
• Trauma or surgery to pelvis or spine
• Depression or stress
• Drugs: antihypertensives, antidepressants,
hormones
What are the causes of ED?
• IMPOTENCE
– Inflammatory – prostatitis
– Mechanical – Peyronie’s disease
– Psychological – depression
– Occlusive vascular factors – arteriogenic, PVD
– Trauma – pelvic fracture
– Extra factors – pelvic surgery/RRP – neuro (cavernosal
nerve)-vascular (to cavernosa) injury
– Neurogenic – stroke/spinal cord defect
– Chemical – alcohol, smoking, drugs (diuretics/anti-HT)
– Endocrine – DM
Aetiological Causes
Psychogenic
• anxiety
• depression
Endocrine
• hormonal deficiency
Neurogenic
• surgery or trauma to
pelvis or spine
• diabetes mellitus or
alcohol
Aetiological Causes
Arteriogenic
• hypertension, smoking, D.M.
• hyperlipidaemia
• surgery or trauma to pelvis
Venous
• functional impairment of the
veno-occlusive mechanism
Drugs
• antihypertensives,
antidepressants
What medication asso with ED?
• Diuretics & antihypertensive
• Antidepressant
• Anti-anxiety
• Anti-epileptic drugs
• Pakinson’s disease med
• Antihistamines
• NSAIDs
• Antiarrhythmics
• H-2 blockers
• Muscle relaxants
• Prostate cancer medication
• Chemotherapy
What is the difference between
psychogenic vs organic causes?
• Psychogenic
– Sudden onset
– Situational
– Morning erection +ve
– Rigidity of morning erection
– Other psychological
complaints
– Spouse relationship
– Abnormal sexual
development
• Organic
– Gradual onset
– Always
– Morning erection –ve
– Chronic medical illness
(DM, HT, IHD)
– Pelvis trauma / surgery
– Endocrine / Neurological
disease
– Recreational drugs
– +/- Loss of libido
– +/- Reduced size of penis
DiagnosisDiagnosis
Scenario
• M/70
• HT/DM
• Worsening erectile function for 1 year
• Affecting relationship with 30-year-old wife
What is the specific history for ED?
• Sexual
– Onset (sudden or gradual)
– Duration, severity – IIEF
– Early morning erections?
– Loss of libido, sexual relationship issues (different partners)
– Previous treatment
– Patient’s expectation
• Medical and surgical
– Hypertension; cardiac disease; peripheral vascular disease; diabetes
mellitus; endocrine or neurological disorders; pelvic surgery,
radiotherapy, or trauma
• Drugs
– Antipsychotic drugs, anti-anxiety drugs, diuretics, chemotherapy, etc
• Psychosocial
– Anxiety, depression
• Social
– Smoking, alcohol consumption
What is IIEF?
• IIEF (International Index of Erectile
Function) 1997
• For assessment of intensity of ED
• Best description of own situation in last 6
months of sexual activity
What is IIEF?
• Five domains
1. Erectile function
2. Orgasmic function
3. Sexual drive
4. Intercourse
satisfaction
5. Overall satisfaction
• Short form: IIEF-5
– Scoring 1-5 for each, Total
25
– No ED: 22-25
– Mild ED: 17-22
– Mild to moderate: 12-16
– Moderate ED: 8-11
– Severe ED 1-7
What is IIEF-5?
What is the physical
examination?
• Height , weight BMI , BP
• Secondary sexual characteristics to rule out hypogonadism
• Thyroid evaluation
• Cardiovascular system
– LL pulses
• Abdominal
– Waist cricumference
• Neurological system
– Penile Sensatiion, bulbocarvernosus reflex, LL neurology
• Genital-urinary system
– Penile deformity, phimosis, Peyronie's plaques
– Testicular size, consistence and mass
– DRE: anal tone, prostate
What is the Ix?
• L/RFT
• PSA
• Fasting glucose
• Lipid profile
• TFT
• Hormone profile
– Testosterone
– Prolactin
• Urinalysis
Why should ED be investgiated
ED is associated with other morbidities in 20%
• DM
• Occult cardiac disease
• Dyslipidaemia
• Endocrine disorders: hypogonadism
• CVA – 50%
Cardiovascular assessmentCardiovascular assessment
What is cardiac evaluation in
ED?
• If happened to men having symptomatic
coronary artery disease ( CAD ), ED
precedes 55 – 65% CAD by 3 – 4 years
• Sex energy expenditure is equal to
15min walk or climbing 2 flights
• Absolute risk that sex can trigger MI is 1-
2 per million
Princeton II consensusPrinceton II consensus
ED as warning sign of
vascular / cardiac disease
What is Princeton II
consensus?
What is Princeton II
consensus?
What is Princeton II
consensus?
• Low risk
– Controlled HT, NYHA 1, <3 risk factors for IHD, stable
angina, uncomplicated past MI
• Intermediate risk
– >/= 3 risk factors, NYHA 2, <6 weeks from MI
• High risk
– High risk arrhythmia, unstable angina, recent MI (<2
weeks), HOCM, NYHA 3-4, uncontrolled HT
What are the indications for
specific diagnostic tests?
• Primary erectile disorder (not caused by organic
disease or psychogenic disorder)
• Young patients with a history of pelvic or perineal
trauma
• Patients with penile deformities that might require
surgical correction, e.g. Peyronie’s disease, congenital
curvature
• Those unresponsive to medical therapy
• Complex psychiatric or psychosexual disorders
• Complex endocrine disorders
• Medicolegal reasons, e.g. implantation of penile
prosthesis, sexual abuse
What are the special Test?
1. Nocturnal penile tumescence (NPT)
2. Caverject Trial
3. Vascular imaging
4. Duplex USG
5. Cavernosometry
6. Penile arteriography
What is nocturnal penile
tumescence testing?
• 80% Noctural Penile Tumescence (NPT) occurs during
REM sleep
• Rigiscan device contains 2 rings which are placed
around base and distal penile shaft to measure
tumescence and number, duration, and rigidity of
nocturnal erections
• Should be done on at least two nights.
• A functional erectile mechanism is indicated by an
erectile event of
– at least 60% rigidity recorded on the tip of the penis
– that lasts for 10 min or more
• Gold Standard
– Organics vs Psychogenic
What is nocturnal penile
tumescence testing?
What is intracavernous
injection test?
• A positive test is a rigid erectile response (unable to
bend the penis) that appears within 10 min after the
intracavernous injection and lasts for 30 min
• This response indicates a functional and rule out
veno-occlusive dysfunction, although co-exist with
arterial insufficiency
• If inconclusive as a diagnostic procedure and Duplex
ultrasound of the penile arteries should be requested
• Assisted in Ix including Duplex USG / DICC / penile
arteriogram
• Positive test shows that a patient will respond to the
intracavernous injection program
What is duplex ultrasound of
penile arteries?
• Assess cavernosal arterial inflow to corpora
cavernosa
• Normal value
– Peak systolic blood flow >35 cm/s
– End diastolic velocity <5 cm/s and
– Resistance index >0.8
• Further vascular investigation is unnecessary when
a Duplex examination is normal
• Name this
investigation (1)
• In investigating
what condition is
this used? (1)
• What is the
finding shown? (1)
Q27
• Pharmacologic cavernosography (1)/
Dynamic infusion cavernosgraphy &
cavernosometry (DICC) (1)
• ED (1)
• Venous leakage along pelvic veins
suggestive of veno-occlusive dysfunction
(1)
Cavernosography
• Indication:
1. evaluate venous problems in men with ED
2. Investigation of priapism (high flow)
3. Assessment of penile fractures/injury to
assess cavernosal damage
4. Assessment of Peyronie’s disease (rarely
used)
• Contraindication:
– Hx of contrast allergy
Carvernosography
• Two 19–22 G butterfly needles inserted into the corpora
• 60-100ml Omnipaque or urograffin infused slowly to
obtain penile pressure 90mmHg
• If penis not erection , contrast leakage
• Fluoroscopy: AP , Rt, Lt oblique view
• Normal: no contrast visualized outside the 2 corpora
cavernosa
• Abnormal: Contrast leakage or significant curvature
• Patient asked to squeeze penis for 5min to ensure
complete emptying
• Advantage: more sensitive and accurate
compare to doppler USG for venous
leakage
• Disadvantage:
– Invasive
– Can be painful
– Risk of infection
– Contrast related fibrosis within corpora
– Risk of priapism
Dynamic Infusion Cavernosometry &
Cavernosography
• 4 phases
– Injection of vasoactive agents (alprostadil,
Bimix, Trimix) into one corpus cavernosum to
relax the corporeal smooth muscles
– Pharmacologic cavernosometry (infusing the
penis with heparinized saline whilst monitoring
the intracavernosmal pressure)
– Cavernosal artery systolic occlusion pressure
(CASOP) reached as intracavernosal pressure
drops
– Pharmacologic cavernosography (infusing
contrast into the corporeal tissue and
obtaining radiographic images of the penis
and perineum to see if there is venous
leakage
• Normal:
– A gradient between the CASOP and the
brachial artery pressures of <35mmHg
– an equal pressure between the right and the
left cavernous arteries
Venous leakage:
-Inability to occlude systolic pressure
-Large gradient between CASOP & brachial
systolic pressure
-Rapid drop of intracavernosal pressure
upon stopping of infusion
What is the indication of arteriography
and dynamic infusion cavernosometry or
cavernosography?
• Arteriography and DICC: dynamic infusion
cavernosometry or cavernosography
should be performed only in patients to
rule out venous leakage who are being
considered for vascular reconstructive
surgery
TreatmentTreatment
Treatment of Erectile Dysfunction
• Treat underlying disease to preserve health
• Elimination of modifiable risk factors
• Disorders that need treatment as part of ED
management
- relationship conflict
- depression, psychogenic ED
- hypogonadism, hyperprolactinaemia
Treatment of Erectile Dysfunction
• Cardiovascular status of the patient
• Is the patient able to resume sexual activity?
- if not, cardiovascular assessment and
intervention may be appropriate
• Patient and partner choices play important
role in identifying successful treatment
What are the treatment
options?
• First-line therapies
– Lifestyle modification,
psychosexual therapy
– PDE5i
• Second-line therapies
– Intraurethral injection of
alprostadil
– Intracavernosal injection of
alprostadil
– Vacuum constriction
devices
• Third-line therapy
– Surgical implantation of
prosthesis
– Penile vascular
reconstruction
What is the lifestyle
modification to improve ED?
• Smoking
• Alcohol
• HT, DM
• Obesity (BMI), exercise
• A multicentre, randomised study
– In obese men with moderate ED compared 2 years of intensive
exercise and weight loss
– Significant improvements in body mass index (BMI) and physical
activity scores, as well as in erectile function
• Esposito K, et al. Effect of lifestyle changes on erectile dysfunction in obese men: a
randomized controlled trial. JAMA 2004 Jun;291(24):2978-84
What is pyschosexual
therapy?
• Identify and treat underlying psychological
diseases
• Provides information and treatment in the
form of sex education, instruction on
improving partner communication skills,
cognitive therapy, and behavioral therapy
What is the general efficacy of
various treatment?
• PDE5i - 80% success
– DM 60%, NSRRP 70%
• ICI - 90%
• VCD - 90%
• MUSE - 50%
Phosphodiesterase type-5Phosphodiesterase type-5
(PDE5) inhibitors(PDE5) inhibitors
What is Phosphodiesterase
type-5 (PDE5) inhibitors?
• sildenafil (Viagra) – half-life 4h
• tadalafil (Cialis) – half-life 17.5h
• vardenafil (Levitra) – half-life 4h
• All have similar effect and outcome
• Enhance cavernosal smooth muscle relaxation and erection by
blocking the breakdown of cGMP to 5GMP. Sexual stimulus is still
required to initiate events
• Make sure no Contraindications
– Concomitant use of nitrates
– Hx of retinitis pigmentosa
– Princeton high risk group
– Severe liver function impairment
• IC50 is the concentration of drug required to produce 50% inhibition
of target enzyme. Vardenafil has the lowest IC50 0.7nM compared
to 0.9 for tadalafil and 3.5 for sildenafil
Character of PDE5-i
Sildenafil
(Viagra)
Vardenafil
(Levitrat)
Tadalafil
(Cialis)
Onset of action 15 min -1hr 15min -1hr 15min -2 hr
Effect of food Reduced absorption
with fatty food
Reduced absorption
with fatty food
NONE
Dosage 20,50,100mg 5,10,20mg 5,10,20mg
Side effects Headache,
dyspepsia, facial
flushing,
blurred/blue vision,
backache, myalygia
Headache,
dyspepsia, facial
flushing,
blurred/blue vision,
backache, myalgia
Prolong QT
Headache,
dyspepsia, facial
flushing , blurred/
blue vision ,
backache, myalgia
Contraindications Nitrates Nitrates, anti-
arrhythmics
Nitrates
How to counsel patient for PDE5-i?
• No data comparing the efficacy for sildenafil,
tadalafil and vardenafil
• Choice of drug will depend on the patient
preference
– Unplanned sex: tadalafil
– Planned sex: Sildenafil & verdenafil
• Taken at least 30min to 1hr before sex, with
empty stomach
• Medication need appropriate sexual stimulation
• Explained potential side effect
Adverse effect
• Headache (20%)
• Flushing (15%)
• Dyspepsia (10%)
• Rhinitis (6%)
• Blurred/ blue vision (6%) (Chromatopsia)
– Cross activity with PDE6 (retinal phototransduction
enzyme)
• Myalgia and back pain (5%)
• Dizziness (5%)
• Priapism (rare)
What to do if failed PDE5-i
• 20% do not respond to any PDE5 inhibitors
• Can change to another if one ineffective
• Exclude fake drug
• Should at least try 4 times for at least 2 drug with maximal dosage
before considering failure
• Re-education:
– Dose , timing of med
– Alcohol, interaction with fatty food
– Adequate sexual arousal
– Try few more times
• Check testosterone: make sure not hypogonadal
• Addition testosterone if hypogonadal:
– General improvement in sexual function
– Improved erection
– Enhanced responsiveness of PDE5-I
• Lipitor (Atorvastatin) improve response to sildenafil (Hermann JSM
2005)
What is non-arteritic anterior
ischaemic optic neuropathy?
• FDA alert 7/2005: small number of men had lost of eyesight
after taking Levitra, viagra and cialis. This non arteritic ischemic
optic neuropathy cause sudden painless loss of eyesight
because blood flow to the optic nerve is blocked. (disc edema)
• It is not known whether these drugs cause NAION, as the
condition also occur in men not taking such drugs
• High risk patients for NAION include
– Over 50
– DM
– Hypertension
– High cholesterol
– Smoker
– Certain eye problem
How about Cardiovascular safety?
• No increase in myocardial infarction rates
• Nitrates are totally contraindicated with PDE5 inhibitors
– cGMP accumulation and unpredictable falls in blood
pressure
– If PDE5I is taken and the patient develops chest pain,
nitroglycerine must be withheld for at least 24 hours
for viagra and levitra, and for at least 48 hours for
cialis
What is the evidence of Changing
the PDE5 inhibitor in non-
responder?
• A randomized, open-label, crossover trial
comparing sildenafil and tadalafil
• Some patients might respond better to
one PDE5 inhibitor than to another and
vice versa
• Might be explained by variation in drug
pharmacokinetics
– Eardley I, et al. Factors associated with preference for sildenafil citrate and tadalafil for
treating erectile dysfunction in men naive to phosphodiesterase 5 inhibitor therapy: post hoc
analysis of data from a multicentre, randomized, open-label, crossover study. BJU Int 2007
Jul;100(1):122-9
What is the evidence of regular dosing of
PDE5 inhibitor in non-responder?
• No randomized trials to support this
intervention
• Although tadalafil is licensed for daily
dosing at a dose of 2.5 mg and 5 mg,
neither sildenafil nor vardenafil are
licensed for use in this way
What is the difference between on-
demand and chronic use of PDE5
inhibitors?
• Double-blind, placebo-controlled,
multicentre, parallel-group study
• Mild-to-moderate ED randomised to
receive once-daily vardenafil 10 mg plus
on-demand vardenafil
• Once-daily dosing does not offer any
sustainable effect after cessation of
treatment
What is the evidence of PDE5
inhibitors in post-RRP?
• Early use of a high dose of sildenafil after
RP is associated with the preservation of
smooth muscle within the human corpora
cavernosa
• The response rate to sildenafil treatment
for ED after RP in different trials up to 70%
among those who underwent bilateral
NSRP and up to 15% among those who
underwent non-NSRP
• Daily sildenafil also resulted in a greater
return of spontaneous normal erectile
function post RP compared to placebo
following bilateral nerve-sparing RP in
patients who were fully potent before
surgery
What is the evidence of PDE5
inhibitors in post-RRP?
• A randomized, double-blind, multicentre
study
• Compared on- demand and nightly dosing
of vardenafil in men with ED following
bilateral NSRP
• Vardenafil was efficacious when used on
demand, supporting a paradigm shift
towards on-demand dosing
Montorsi F, et al. Effect of nightly versus on-demand vardenafil on recovery of erectile
function in men following bilateral nerve-sparing radical prostatectomy. Eur Urol
2008;54(4):924-31
What is the precaution with
anti-hypertensive drugs?
• Small additive drops in blood pressure,
which are usually minor
• Even when the patient is taking several
antihypertensive agents
How about Alpha-blocker
interactions?
• Viagra: 50, 100mg not to be taken within a 4
hour window of an alpha blocker
• Levitra: safe to use with tamsolusin. (label
changed in US from previous total
contraindication with alpha blockers)
• Cialis: safe to use with tamsolusin 0.4mg
ApomorphineApomorphine
What is Apomorphine?
• Dopamine receptor agonist
• Sublingual (Uprima SL)
– Erections are achieved within 20 min
• Efficacy rates (erections sufficient for intercourse) range from 30%
to 55%)
• Acts centrally on dopaminergic receptors in the paraventricular
nucleus of the hypothalamus to enhance and co-ordinate the effect
of sexual stimuli
• Adverse effects: nausea; headache; dizziness
• Apomorphine is not contraindicated in patients taking nitrates or
antihypertensive drugs
• Used in patients with certain contraindications for the use of PDE5
inhibitors, e.g. nitrates
• Multiple daily dosing possible: once / 8 hours
Intraurethral therapyIntraurethral therapy
What is the mechanism of
prostaglandin E?
• Prostaglandin E binds to PGE receptor,
activating adenylate cyclase which
converts ATP to cAMP
• cAMP activate PKA which brings about
the relaxation of the smooth muscle
directly without NO pathway
What is intraurethral therapy?
• Alprostadil
• Synthetic prostaglandin E1 (PGE1) pellet administered
into the urethra via a specialized applicator
• Once inserted, the penis is gently rolled to encourage
the pellet to dissolve into the urethral mucosa, from
where it enters the corpora
• Efficacy – 50%
• Side-effects: penile pain; priapism; local reactions
Intracavernosal therapyIntracavernosal therapy
What is Intracavernosal
therapy?
• Alprostadil
• Increase cAMP within corporal smooth muscle
 relaxation of SMC
• Right angle at lateral mid-penile shaft
• Efficacy rates for intracavernous alprostadil -
90%
• Adverse effects: pain; priapism; haematoma
• Contraindications: men at risk of priapism (1%)
and men with bleeding disorders
What is Intracavernosal
therapy?
What is Intracavernosal
therapy?
Locate the area of injection. Wipe off with an alcohol swab. Grasp the head of the
penis, not the skin. Position the penis along your inner thigh. Maintain traction on the
head after cleaning the side of the penis.
What is Intracavernosal
therapy?
• Most long-term injection users can switch
to sildenafil despite underlying
pathophysiology
• Almost one-third of long-term
intracavernous injections users who
subsequently responded also to sildenafil
preferred to continue with an
intracavernous injection programme
What is combination therapy?
• Papaverine – combination therapy today
due to its high incidence of side-effects as
monotherapy
• Phentolamine - in combination therapy to
increase efficacy. As monotherapy, it
produces a poor erectile response
What is combination therapy?
• Papaverine plus phentolamine plus alprostadil
have never been licensed for ED
• Combination had similar side-effects as
alprostadil monotherapy, but a lower incidence
of penile pain due to lower doses of alprostadil
• Fibrosis and priaprism were more common when
papaverine was used. In addition, mild
hepatotoxicity has been reported with
papaverine
What is the action to be taken
with a prolonged erection?
• 19-gauge needle is used to aspirate blood
• If failed, an intracavernous injection of
phenylephrine, starting at a dose of 200
μg every 5 min and increasing to 500 μg if
necessary
Vacuum erection deviceVacuum erection device
Vacuum Constriction Device
• What is this device? (0.5)
• Name one condition it is used for (0.5)
• Name one complication from its use? (1)
• Vaccum constriction device (0.5)
• ED, Peyronie’s disease (0.5)
• Cold penis, difficulty with ejaculation,
bruising, penile numbness (1 max)
Vaccum constriction device
• Erections with these do not use physiological erection pathways
• Plastic cylinder connected directly or by tubing to a vacuum-
generating source (manual or battery-operated pump)
• Penis is engorged by the negative pressure
• Constricting ring is applied to the base to maintain the erection
 should not be left in place > 30 minutes
• Can be used successfully by men with a malfunctioning penile
prosthesis in place
• Used after explanation to prevent shortening
• Disadvantage: cold numb penis, no ejaculation, discomfort in
orgasm
• Patients taking aspirin or warfarin should exercise caution when
using these devices
• Erections satisfactory for intercourse, is as high as 90%, but
decreases to 50-64% after 2 years. Most men who discontinue
within 3 months
Penile prosthesisPenile prosthesis
What is Penile prosthesis?
• Third-line therapy
• Surgical implantation into the corpora to provide penile rigidity
and sufficient erectile size for sexual intercourse
• Two types of prosthesis exist: malleable (semi-rigid) and
inflatable (two- or three-piece)
• Most patients prefer the three-piece inflatable devices due to
the more ‘natural’ erections obtained.
• Two-piece inflatable prosthesis can be a reliable option with
fewer mechanical complications and is easier to implant
• A semi-rigid prosthesis provides a constantly rigid penis and
may be suitable in older patients with infrequent sexual
intercourse with less mechanical failure but erosion and chronic
pain
• AMS 700
• 5YS 90%
• Side-effects: mechanical failure (5% per year); erosions (5%);
infections (2%)
• Reservoir in abdomen
• Pump in scrotum, pair of cylinders implant into penis
• Infection rate may be reduced to 1% by implanting an antibiotic-
impregnated prosthesis
• Infection rate is similar between primary VS revision, DM VS
non-DM
• Staphylococcus epidermidis- commonest
• Mulcahy technique has been described in an attempt to
salvage the situation and insert a new prosthesis at the time of
the infected one. It involves copious wound irrigation with
kanamycin, bacitracin, iodine, hydrogen peroxide, vancomycin
and gentamycin. The success rate is 80% at 3 years
Surgical penileSurgical penile
revascularizationrevascularization
What is surgical penile
revascularization?
• Post-traumatic arteriogenic ED in young patients
• In young patients with pelvic or perineal trauma,
surgical penile revascularization has a 70% long-
term success rate
• The lesion must be demonstrated by Duplex
ultrasound and confirmed by selective internal
pudendal arteriogram
• Corporeal veno-occlusive dysfunction is a
contraindication to revascularization and must be
excluded by DICC
• Vascular surgery for veno-occlusive dysfunction is
no longer recommended because of poor long-
term results
Androgen deficiencyAndrogen deficiency
hypogonadism
• 95% testosterone is produce by Leydig cells
• Serum level peak at 7-8am and lowest at midnight. Clinically
serum testosterone is best estimated in the morning
• Incidence of late-onset hypogonadism: 20% of men over 70
• <10% ED due to hypogonadism
• Serum testosterone is loosely bound to albumin in 50% and
50% to sex hormone binding globulin (SHBG). Whereas 2% is
free. The bioactive testosterone refers to free and albumin
bound serum testosterone (~50%)
• Estimation of the bioavailable testosterone is a more accurate
test than total serum testosterone when investigating
hypogonadism
• SHBG is increase in: ageing , cirrhosis, hyperthyroidism ,
anticovulsants, oestrogen, HIV infection
Approach
• History:
– loss of libido, low mood, lethargy
– Change in sexual function
• Physical examination:
– Loss of muscle mass & hair loss
– regression of secondary sexual characteristics
– softer and smaller testis,
– gynaecomastia (aromatisation of testosterone in fatty
tissue to oestrogen)
• Investigation:
– fasting glucose, lipid,
– total testosterone, free testosterone, FSH and LH
– Oestradiol, TFT & prolactin in selected case
Late-onset male hypogonadism
• Symptom complex resulting from age-
related decline in testosterone level in
men
• Cause:
– Primary: Testicular failure
– Secondary:
• Pituitary or hypothalamic disorder (Kallman’s
syndrome)
• Conbined hypogonadism
Treatment
• Weight loss
• Clomiphene citrate
• Androgen replacement therapy
–CI: polycythemia, fluid retention, Ca prostate
and breast , sleep apnoea, heart failure,
severe renal and liver failure
–Oral – first pass to liver for metabolism > too
fast to have an effect
–Transdermal, subcutaneous, intramuscular
Monitor
• Serum testosterone level
• Clinical signs and symptoms
• Assess bone mineral density
• Adverse effect:
– Excessive rise in hematocrit (>54%)
– Raised PSA or abnormal DRE
– Ance
– Increase oliness of skin
– Gynaecomastia
– Suppression of fertility
– Some testicular atrophy
Testosterone & Ca Prostate?
• No positive correlation btw testosterone
level and Ca Prostate in prospective
epidemiological studies
• Small clinical trails have not shown
increase in clinical prostate cancer in the
testosterone group compared with the
placebo groups
• But no sufficient data to drawn conclusion
Ejaculatory disorder
Ejaculatory disorder
• Hematospermia
• Retrograde ejaculation
• Premature ejaculation
• Ejaculatory failure
Hematospermia
• Commonly seen after prolonged period of sexual
abstience, always resolved spontaneously
• Investigation if beyond several weeks
• History:
– Exclude hematuria
– Recent trauma
– Infection (STD)
– Bleeding disorder
– PMH: TB, Ca prostate
• Three important point need evaluation:
– Patient age
– Duration & recurrence
– Associated hematuria
• Physical examination:
– Blood pressure
– Genital: TB, bead cord vas
– Penis
– DRE: prostate
• Investigation:
– Bld, PSA
– MSU : sterile pyuria
– Urethral swab for younger pt
– Cytology
– FR + RU if slow stream
Cause
• Infection (40%): TB, HIV, CMV
• STD: herpex, chlamydia, ureaplasma
• Prostatitis (30%)
• Post-TRUS + bx
• Prostate cancer
• Urethritis and urethral stricture
• Acquire or congenital cyst of the seminal vesicle
• Systemic disorder: HT, liver disease, lymphoma
Further investigation
• TRUS: reveal abnormalities in 95%
– Prostatic calcification (40%)
– Ejaculatory duct calculi (40%)
– Dilated ejaculatory duct (30%)
– Ejaculatory duct cyst (10%)
– BPH (30%)
– Dilated or clacified SV (20%)
– Mullerian duct remnants (7%)
• Treatment:
– Reassurance
– Antibiotics ?
Retrograde ejaculation
• History:
– Low ejaculate volume
– Post- ejaculate urine cloudy
• Investigation : post-ejaculate urine examination for
sperm
• Cause :
– Post- RPLND
– DM
– Bladder neck surgery, TURP
– Trauma
– Alpha- blockers
– Urethral stricture
– Spinal cord injury
Treatment
• Medication: to close the bladder neck
– Sympathomimetic : pseudoephedrine & ephedrine
– TCA: imipramine
– Efficacy : 50%
• IVF:
– Sperm retrived from alkalinised post-ejaculate urine
– Fertilization rate: 50%
• How to alkalinise urine?
– Sodium bicarbonate: 1gm at night before & 1mg in
the morning of sperm collection
– Or use Liverpool solution : NaCl + NaHCO3
– Empty bladder before masturbation
– Obstain post-ejaculated urine & send to lab ASAP
Premature ejaculationPremature ejaculation
Premature Ejaculation
• Most important point:
1. Short ejaculatory latency time
2. Lack of ejaculatory control
3. Decreased satisfaction with sexual
intercourse
• Interpersonal distress
• Negative man’s self-esteem
• Reduced sexual function and QOL
Patrick DL et al ,J Sex med 2005
Giuliano F et al, Eur Urol 2008
Rowland DL et al, J Urol 2007
DSM-IV-TR 2000
• Persistent or recurrent ejaculation with
minimal sexual stimulation;
–Before, on , or shortly after penetration
–Before the person wishes it;
• Must also cause marked distress or
interpersonal difficulty;
• Cannot be due exclusively to the direct
effects of a substance.
The International Society for Sexual
Medicine (ISSM)
• The first evidence-based definition
• ‘Premature ejaculation is a male sexual
dysfunction characterized by ejaculation which
always or nearly always occurs prior to or within
about one minute of vaginal penetration; and
inability to delay ejaculation on all or nearly all
vaginal penetrations; and negative personal
consequences, such as distress, bother,
frustration and/or the avoidance of sexual
intimacy
Classification
• Lifelong condition
• Acquired condition
• Natural variable PE
• Premature –like ejaculatory dysfunction
Cooper AJ et al, J Sex Maritla Ther 1993
Waldinger MD et al, Drugs 2007
What is the prevalence?
• Major problem in assessing the
prevalence of PE is the lack of an
accurate (validated) definition
• The most common male sexual
dysfunction, with prevalence rates of 20-
30%
• Prevalence of PE is not affected by age
Etiology
Psychogenic:
• Anxiety
• Early sexual
experience
• Infrequent sexual
intercourse
• Poor ejaculatory
control technique
• Negative conditioning
Biological cause:
• Penile
hypersensitivity
• Hyperexcitable
ejaculatory reflx
• Endocrinopathy
• Genetic
predisposition
• 5HT- receptor
dysfunction
Neurophysiology of ejaculation
Neurophysiology
• Ejaculatory control centers in spinal cord
• Received peripheral afferents and
supraspinal influences
• Coordinate sympathetic , parasympathetic
and somatic outputs to pelviperineal
structrues
5-HT neurons
• Activation of 5-HT1A autoreceptors  decrease
5-HT release by presynaptic neurons (-ve
feedback)
• Activation of 5-HT1A decrease ejaculatory
latency
• Activation of postsynaptic 5-HT2C or 5-HT1B
receptors prolongs ejaculatory latency
• PE may be due to imbalance btw 5-HT1A
(hypersensitivity) and 5-HT2C or 5-HT1B
(hyposensitivity)
• Increase central 5-HT  delay ejaculation
What is the approach to PE?
• History and physical examination
– Intravaginal ejaculatory latency time (IELT)
• Clinical use of self-estimated IELT is adequate, stopwatch-
measured IELT is necessary in clinical trials
– Patient-reported outcomes (PROs) have the potential to identify
men with PE
• Patient-reported outcomes (PROs) have the potential to
identify men with PE
• Further research is needed before PROs can be
recommended for clinical use
– Duration time of ejaculation, degree of sexual stimulus, impact
on sexual activity and QoL, and drug use or abuse
– It is also important to distinguish PE from ED
– ED develop secondary PE caused by the anxiety
– Examination of the vascular, endocrine and neurological
systems
• Routine laboratory or neurophysiological tests are not
recommended
Measurement of response
• Intravaginal ejaculatory latency time
(IELT)
– Time between vaginal intromission &
ejaculation
• Perceived controlled over ejaculation
• Improvement of personal distress
What are behavioural therapy?
• ‘Stop-start’ programme developed by Semans
• ‘Squeeze’ technique, proposed by Masters and Johnson
• Masturbation before anticipation of sexual intercourse
• Success rates of 50-60% in short term
• Time intensive, require the support of a partner and can
be difficult to do
• Recurrence is likely after treatment cessation
Pharmacotherapy for PE
• Anti-depressant:
–TCA: Clomipramine
–SSRIs: paroxetine, fluoxetine, sertraline, etc
• Phosphodiesterase-5 inhibitors (PDE-5i)
• Tramadol
• Topical agents: lidocaine/prilocaine
SSRI
• Increase synaptic 5-HT concentration via blockade of 5-HT
transporters
• Paroxetine (20-40mg), Clomipramine (10-50mg) or fluoxetine (20-
40mg)
• Meta-analysis: Paroxetine produce strongest delay in ejaculation
• Daily txn, effect start on 2 week
• Need to withdrawn gradually over 4 week (except fluoxetine)
• SE:
– Psychiatric and neurological
– Dermatological reaction
– Anticholinergic SE
– Change in body weight
– Cognitive impairment
– Drug-drug interactions
– Sexual SE: ED and loss of libido
SSRI discontinuation syndrome
• Especially in paroxetine
• 1-3 days after drug discontinuation
• Median duration: > 1 week
• Reversible when SSRI reintroduced
• Dizziness, nausea and emesis, headache,
gait instability, lethargy, agitation , anxiety
and insomnia
Black K et al, J Psy Neurosci 2000
Haddad P et al, J psychopahrmacol 1998
Tamam L et al, Adv ther 2002
Serotonin syndrome
• SSRI with long half-lives
• Interactions with agents that enhance 5-
HT CNS activity
• Myoclonus, hyper-reflexia, sweating,
shivering , lack of coordination and mental
status changes
Nelson EB et al, J Clin Psychiatry 1997
Lane R et al, J Clin Psychopharmacol 1997
Dapoxetine
• New agents under development
• Rapid onset (1.29hr) and short half-live
(1.49hr)
• On-demand dapoextine 30 or 60mg
significantly improved outcome vs placebo
• IELT increase 3.6x from baseline
• SE: nausea, diarrhoea, headache,
dizziness and insomnia
Pryor JL et al, Lancet 2006
PDE-5i
• Results has been conflicting
• No pharmacological rationale
• Paroxetine + sildenafil vs paroxetine:
increased IELT and satisfaction but with
more SE (headaches and flushing)
Salonia A et al, J Urol 2002
• No effect in men without coexiting ED,
cause decrease in post-ejaculatory
refractory period
Chen J et al, Urology 2002
Tramadol
• Centrally acting synthetic opioid
• Inhibit nor-adrenaline and serotonin
reuptake
• Rapidly absorbed and eliminated
• Increased IELT, sexual satisfaction and
ejaculatory control vs placebo (p<0.05)
Safarinejad MR et al, J Clin Psychopharmacol 2006
Salem EA et al, J Sex Med 2008
Topical agents
• Topical lidocaine/prilocaine cause
desensitization
• Increase mean IELT by 2.4x vs placebo
(p<0.01)
Dinsmore WW et al, BJU Int 2006
• SE: local numbness(12%), loss of erection
• Severance Secret cream: increase IELT
and sexual satisfaction vs placebo
Choi HK et al, Urology 2000
Choi HK et al, Int J impot Res 1999
Conclusion
• PE is an under-treated condition due to
lack of understanding of its cause and
potential therapy, and because of its
sensitive nature
• 5-HT has been implicated as a key
mediator of ejaculatory control
Conclusion
• Available therapy include off-label use of
SSRIa and PDE-5i , as well as topical
anaesthetics
• New on-demand agents like tramadol and
dapoxetine are currently under evaluation
• Role of other central neurotransmitter as
future targets to delay ejaculation needs
further investigation
Ejaculatory failure
• Cause: Post SCI, RPLND , psychogenic
• History: what level , bowel and bladder fxn
• Investigation:
– SA: azzospermia
– Post-orgasmic urine  no fructose
• Treatment: Electro-ejaculator
– Seager electro-ejaculator
– Rectal probe to stimulate perirectal, periprostatic sympathetic
nerves
– May require GA
– Watch out for autonomic dysreflexia in above T6 lesion
– Sperm: poorere quality & mobility
• Alternative: sperm retrieval technique (pregnancy rate
70%)
Peyronie’s diseasePeyronie’s disease
What is Peyronie’s disease?
• Fibrous plaque within tunica albuginea of
penis
• associated with DM, antiepileptic drugs
and beta blockers
• Curvature, penile pain or shortening
• Erectile dysfunction
What is the cause?
• Trauma to tunica albuginea
• Wound healing > excessive fibrotic plaque
• Dorsal plaque more common
• Penile curvature as corpus cavernosum can’t
lengthen fully on erection limited by plaque
• Associated with Dupuytren’s contracture 30%
• Incidence <5%, men aged 40-70 years
What is natural course of the
disease?
• Active phase 6 months, painful erection
with changing deformity
• Quiescent phase 9-12 months stable
deformity, painless
• Natural Hx over 18 m
– 13% improved
– 40% stable
– 47% progress
How to make diagnosis?
• By history and P/E
– History
• Disease Duration
• Pain
• Penile deformity –angle, direction
• Stability
• Penile length
• Erections , able to penetrate
• IIEF
• Risk factor for ED
– P/E
• Assess degree of curvature by 1 photogragh, 2 IC PGE1
• Exam for plaques, location, size
• Penile length, stretched & flaccid state
• Extremeties for Dupuytren’s contracture
What is the treatment?
• Early disease <3m consider medical Tx /
injection / ESWL, low successful rate
– Oral vit E
• 200mg tds for 3 months
• In a randomized trial vitamin E has been shown to
improve pain in 75% of patients and improves the
deformity in 10%
– Oral colchicine x3m (limited evidence for efficacy)
– Intralesional verapamil, steroid x 6m
Photograph taken during a procedure
• What is being done? (1)
• What procedure is this? (0.5)
• Name 3 complications from this procedure (0.5 each)
Q45
• Artificial erection from injection of saline into
corpus cavernosum (1)
• Correction of penile curvature eg. in Peyronie’s
disease (0.5)
• Shortening of penis, erectile dysfunction,
deformity recurrence, palpable suture through
penile shaft skin, altered/decreased penile
sensation (0.5 each, total 1.5)
Is ESWL useful?
• Initiating an inflammatory reaction thru
direct damage to plaque and result in
plaque resorption
• No study has demostrate any
improvement in plaque size or curvature
• NICE do not recommend
What are the indications for
Surgery?
• Disease present for at least 12m
• stable for at least 3m
• Deformity makes intercourse difficult
• Quality of erection important
• ED > ? Prosthesis
Surgery: Penile shortening
• Indicated in pt: no ED , <60degree curvature, no
hourglass demormities or hinge effect
• Must warn pt of Penile shortening effect
• Nesbit
– Penis degloved via circumglandular incision
– Artificial erection with NS
– ellipitcal incision: 1mm for 10 degree deformity
curvature on convex side
– Complication: all penile shortening, 1% ED rate,
recurrence of deformity
– Success rate 80%
Surgery: Penile maintaining
• Lue’s procedure
– By incising the plaque and interposing a graft (fascia lata / vein graft/ Gortex
graft)
– Do cause penile shortening but not to the extent that corporal plication
– More ED – 15%
– Not recommended for complete excision of the plaque due to compromising veno-occlusive
mechanism and causing ED
• Surgery in general
– Success rate: 80%
– Risk: bleeding, infection , bruising
– Loss of 1cm in 26%
– ED in 15%
– Recommend penile traction device or penile rehab with PDE5-i
• About 10% of patients will subsequently require circumcision due to secondary
phimosis
When is prothesis required?
• Penile prosthesis is indicated in patients
with both Peyronie’s disease and severe
ED
• After insertion and inflation of penile
prosthesis, the penis is bent in opposite
direction to break the plaque (modelling)
– 90% successful rate
PriaprismPriaprism
Priapism
• Definition:
– Persistent erection > 4 hours
– Not related to sexual desire
• Two age group:
– 5-10 yo
– 20-50 yo
3. Stuttering priaprism – repeated, shorter self-
Presentation
• FOUR Main questions:
1. Duration of erection > 4 hours?
2. Painful / non painful ? (Ischemic vs nonischemic)
3. Previous history of priapism
4. Predisposing factors
• Physical examination:
– Rigid corpora cavernosa
– Flaccid Corpus spongiosum and glans penis
Investigation
• Blood: CBP, Hb/electrophoresis (SSD)
• Urine: C/ST , toxicology
• Penile blood gas:
– Aspirate blood directly from either corpora
• Duplex USG of carvernosal arteries:
– Ischemic ( inflow low or absent)
– Non ischemic (inflow normal or high)
• Penile pudenal arteriography: not readily available
Appearance pH PO2
(mmHg)
PCO2
(mmHg)
Low flow Dark red <7.25 <30 >60
High flow Bright red = 7.4 >90 <40
Treatment
• Conservative
• Medical
• Minimally invasive
• Surgical treatment
• Always warn patient about the possibility
of impotence due to cavernosal fibrosis
Doppler USG can differentiate high or low flow priaprism
Treatment for Priapism
• Cavernosal aspiration successful rate: 1/3
• Distal shunt
– Winter (large biopsy needle , corporo-glanular)
– Ebbehoj (Scalpel, corporo-glanular)
• Lue’s modification “T-Shunt” (scalpel, corporo-glandular)
– El-Ghorab: piece of tunic albuginea excised at tips of coprora via a
dorsal transverse incision just distal to corona
• Proximal shunt:
– For failed distal shunt or severe distal penile edema
– 80% successful rate, but ED > 90%
– Quackels /Sacher (corporo-spongiosal)
– Grayhack (corporo-saphenous)
• Supra-selective embolisation of common penile artery – successful rate 80%
– Absorbable materials like clots and gel cause less ED than coils or
permanent chemical
El-Ghorab
Quackels /Sacher
Grayhack
A procedure for a urologic emergency is about to be
performed
• What is the name of this procedure? (2)
• What is it used for? (1)
Q20
• Ebbehoj shunt, a type of distal
cavernoglandular shunt (2, 1 mark for
mentioning just “shunt” without name)
• Ischemic priapism not responsive to
injectional medical treatment (1)
How about high-flow
priaprism?
• Not a urological emergency
• Duplex USG can confirm diagnosis
• Selective internal pudendal embolisation, better with
absorbable material including clots or gels
• Successful rate up to 80%
• If failed > open exploration and direct ligation
Erectile Dysfunction [Dr. Edmond Wong]

Weitere ähnliche Inhalte

Was ist angesagt?

Erectile dysfunction (ed)
Erectile dysfunction (ed)Erectile dysfunction (ed)
Erectile dysfunction (ed)Ratheesh R
 
Raising Hope for Fading Manhood
Raising Hope for Fading ManhoodRaising Hope for Fading Manhood
Raising Hope for Fading ManhoodSiewhong Ho
 
Erectile dysfunction
Erectile dysfunctionErectile dysfunction
Erectile dysfunctionUdr Farouk
 
Erectile Dysfunction Symptoms And Treatment
Erectile Dysfunction Symptoms And TreatmentErectile Dysfunction Symptoms And Treatment
Erectile Dysfunction Symptoms And TreatmentManas Das
 
sexual-arousal-disorders
 sexual-arousal-disorders sexual-arousal-disorders
sexual-arousal-disorderscjsmann
 
Premature Ejaculation
Premature EjaculationPremature Ejaculation
Premature EjaculationEko indra
 
Premature ejaculation
Premature ejaculationPremature ejaculation
Premature ejaculationJim Badmus
 
erectile dysfunction.pptx
erectile dysfunction.pptxerectile dysfunction.pptx
erectile dysfunction.pptxaditya romadhon
 
Benign Prostatic Hyperplasia BPH [Dr. Edmond Wong]
Benign Prostatic Hyperplasia BPH [Dr. Edmond Wong]Benign Prostatic Hyperplasia BPH [Dr. Edmond Wong]
Benign Prostatic Hyperplasia BPH [Dr. Edmond Wong]Edmond Wong
 
Disorder of male sexual function
Disorder of male sexual function  Disorder of male sexual function
Disorder of male sexual function ANILKUMAR BR
 
Erectile dysfunction and Premature Ejaculation
Erectile dysfunction and Premature Ejaculation Erectile dysfunction and Premature Ejaculation
Erectile dysfunction and Premature Ejaculation Dr. Amit Chougule
 
Female sexual dysfunction
Female sexual dysfunction Female sexual dysfunction
Female sexual dysfunction NITISH SHAH
 
Premature Ejaculation
Premature EjaculationPremature Ejaculation
Premature EjaculationGAURAV NAHAR
 
Male Sexual Dysfunction
Male Sexual DysfunctionMale Sexual Dysfunction
Male Sexual DysfunctionSebastian
 

Was ist angesagt? (20)

Erectile dysfunction (ed)
Erectile dysfunction (ed)Erectile dysfunction (ed)
Erectile dysfunction (ed)
 
Raising Hope for Fading Manhood
Raising Hope for Fading ManhoodRaising Hope for Fading Manhood
Raising Hope for Fading Manhood
 
Erectile dysfunction
Erectile dysfunctionErectile dysfunction
Erectile dysfunction
 
Erectile Dysfunction Symptoms And Treatment
Erectile Dysfunction Symptoms And TreatmentErectile Dysfunction Symptoms And Treatment
Erectile Dysfunction Symptoms And Treatment
 
sexual-arousal-disorders
 sexual-arousal-disorders sexual-arousal-disorders
sexual-arousal-disorders
 
Premature Ejaculation
Premature EjaculationPremature Ejaculation
Premature Ejaculation
 
Overactive bladder
Overactive bladderOveractive bladder
Overactive bladder
 
Premature Ejaculation
Premature EjaculationPremature Ejaculation
Premature Ejaculation
 
Urinary Incontinence
Urinary IncontinenceUrinary Incontinence
Urinary Incontinence
 
Premature ejaculation
Premature ejaculationPremature ejaculation
Premature ejaculation
 
erectile dysfunction.pptx
erectile dysfunction.pptxerectile dysfunction.pptx
erectile dysfunction.pptx
 
Benign Prostatic Hyperplasia BPH [Dr. Edmond Wong]
Benign Prostatic Hyperplasia BPH [Dr. Edmond Wong]Benign Prostatic Hyperplasia BPH [Dr. Edmond Wong]
Benign Prostatic Hyperplasia BPH [Dr. Edmond Wong]
 
Dyspareunia - Anderson Consulting
Dyspareunia - Anderson ConsultingDyspareunia - Anderson Consulting
Dyspareunia - Anderson Consulting
 
Disorder of male sexual function
Disorder of male sexual function  Disorder of male sexual function
Disorder of male sexual function
 
Erectile dysfunction and Premature Ejaculation
Erectile dysfunction and Premature Ejaculation Erectile dysfunction and Premature Ejaculation
Erectile dysfunction and Premature Ejaculation
 
Erectile dysfunction
Erectile dysfunctionErectile dysfunction
Erectile dysfunction
 
Female sexual dysfunction
Female sexual dysfunction Female sexual dysfunction
Female sexual dysfunction
 
Female sexual function dysfunction
Female sexual function dysfunctionFemale sexual function dysfunction
Female sexual function dysfunction
 
Premature Ejaculation
Premature EjaculationPremature Ejaculation
Premature Ejaculation
 
Male Sexual Dysfunction
Male Sexual DysfunctionMale Sexual Dysfunction
Male Sexual Dysfunction
 

Andere mochten auch

Diabetes mellitus and erectile dysfunction by Dr. Mohand Yaghi PgDip (urol) C...
Diabetes mellitus and erectile dysfunction by Dr. Mohand Yaghi PgDip (urol) C...Diabetes mellitus and erectile dysfunction by Dr. Mohand Yaghi PgDip (urol) C...
Diabetes mellitus and erectile dysfunction by Dr. Mohand Yaghi PgDip (urol) C...Mohand Yaghi
 
Erectile Dysfunction Treatment Without Medication or Operation
Erectile Dysfunction Treatment Without Medication or OperationErectile Dysfunction Treatment Without Medication or Operation
Erectile Dysfunction Treatment Without Medication or OperationBetterBlue
 
DIABETES INDUCED ERECTILE DYSFUNCTION
DIABETES INDUCED ERECTILE DYSFUNCTIONDIABETES INDUCED ERECTILE DYSFUNCTION
DIABETES INDUCED ERECTILE DYSFUNCTIONKishore Krishn
 
Ueda2015 d erectile dysfunction patients_dr.khaled mohy
Ueda2015 d erectile dysfunction patients_dr.khaled mohyUeda2015 d erectile dysfunction patients_dr.khaled mohy
Ueda2015 d erectile dysfunction patients_dr.khaled mohyueda2015
 
Erectile dysfunction in diabetes
Erectile dysfunction in diabetesErectile dysfunction in diabetes
Erectile dysfunction in diabetesPeninsulaEndocrine
 
Yousry Endothelial Vs Erectile Dysfunction
Yousry Endothelial Vs Erectile DysfunctionYousry Endothelial Vs Erectile Dysfunction
Yousry Endothelial Vs Erectile Dysfunctionguest5a246c
 
Erectile Dysfunction And Tadalafil
Erectile Dysfunction And TadalafilErectile Dysfunction And Tadalafil
Erectile Dysfunction And TadalafilBALASUBRAMANIAM IYER
 
Couple therapy and treatment of sexual dysfunction
Couple therapy and treatment of sexual dysfunctionCouple therapy and treatment of sexual dysfunction
Couple therapy and treatment of sexual dysfunctionGladys Escalante
 
Premature ejaculation
Premature ejaculation Premature ejaculation
Premature ejaculation Wong Lei
 
Penile Rehabilitation Program
Penile Rehabilitation ProgramPenile Rehabilitation Program
Penile Rehabilitation ProgramChad Hansen
 
Erectile Dysfunction Treatment
Erectile Dysfunction Treatment Erectile Dysfunction Treatment
Erectile Dysfunction Treatment austinsamo
 
Developing sexuality and sexual health policies in the disability sector
Developing sexuality and sexual health policies in the disability sector Developing sexuality and sexual health policies in the disability sector
Developing sexuality and sexual health policies in the disability sector Werksmans Attorneys
 
Biology 9 1
Biology 9 1Biology 9 1
Biology 9 1Tamara
 
Sexuality Training for Teens or Adults with Developmental Disabilities
Sexuality Training for Teens or Adults with Developmental DisabilitiesSexuality Training for Teens or Adults with Developmental Disabilities
Sexuality Training for Teens or Adults with Developmental DisabilitiesAmber Osborn
 

Andere mochten auch (18)

Diabetes mellitus and erectile dysfunction by Dr. Mohand Yaghi PgDip (urol) C...
Diabetes mellitus and erectile dysfunction by Dr. Mohand Yaghi PgDip (urol) C...Diabetes mellitus and erectile dysfunction by Dr. Mohand Yaghi PgDip (urol) C...
Diabetes mellitus and erectile dysfunction by Dr. Mohand Yaghi PgDip (urol) C...
 
Erectile Dysfunction Treatment Without Medication or Operation
Erectile Dysfunction Treatment Without Medication or OperationErectile Dysfunction Treatment Without Medication or Operation
Erectile Dysfunction Treatment Without Medication or Operation
 
DIABETES INDUCED ERECTILE DYSFUNCTION
DIABETES INDUCED ERECTILE DYSFUNCTIONDIABETES INDUCED ERECTILE DYSFUNCTION
DIABETES INDUCED ERECTILE DYSFUNCTION
 
Ueda2015 d erectile dysfunction patients_dr.khaled mohy
Ueda2015 d erectile dysfunction patients_dr.khaled mohyUeda2015 d erectile dysfunction patients_dr.khaled mohy
Ueda2015 d erectile dysfunction patients_dr.khaled mohy
 
Erectile dysfunction in diabetes
Erectile dysfunction in diabetesErectile dysfunction in diabetes
Erectile dysfunction in diabetes
 
Yousry Endothelial Vs Erectile Dysfunction
Yousry Endothelial Vs Erectile DysfunctionYousry Endothelial Vs Erectile Dysfunction
Yousry Endothelial Vs Erectile Dysfunction
 
Erectile Dysfunction
Erectile DysfunctionErectile Dysfunction
Erectile Dysfunction
 
Erectile Dysfunction And Tadalafil
Erectile Dysfunction And TadalafilErectile Dysfunction And Tadalafil
Erectile Dysfunction And Tadalafil
 
Dapoxetin
DapoxetinDapoxetin
Dapoxetin
 
Couple therapy and treatment of sexual dysfunction
Couple therapy and treatment of sexual dysfunctionCouple therapy and treatment of sexual dysfunction
Couple therapy and treatment of sexual dysfunction
 
Androgens - drdhriti
Androgens - drdhritiAndrogens - drdhriti
Androgens - drdhriti
 
Premature ejaculation
Premature ejaculation Premature ejaculation
Premature ejaculation
 
Classification and Diagnosis of Sexual Dysfunctions
Classification and Diagnosis of Sexual DysfunctionsClassification and Diagnosis of Sexual Dysfunctions
Classification and Diagnosis of Sexual Dysfunctions
 
Penile Rehabilitation Program
Penile Rehabilitation ProgramPenile Rehabilitation Program
Penile Rehabilitation Program
 
Erectile Dysfunction Treatment
Erectile Dysfunction Treatment Erectile Dysfunction Treatment
Erectile Dysfunction Treatment
 
Developing sexuality and sexual health policies in the disability sector
Developing sexuality and sexual health policies in the disability sector Developing sexuality and sexual health policies in the disability sector
Developing sexuality and sexual health policies in the disability sector
 
Biology 9 1
Biology 9 1Biology 9 1
Biology 9 1
 
Sexuality Training for Teens or Adults with Developmental Disabilities
Sexuality Training for Teens or Adults with Developmental DisabilitiesSexuality Training for Teens or Adults with Developmental Disabilities
Sexuality Training for Teens or Adults with Developmental Disabilities
 

Ähnlich wie Erectile Dysfunction [Dr. Edmond Wong]

4. Erectile dysfunction.pptx
4. Erectile dysfunction.pptx4. Erectile dysfunction.pptx
4. Erectile dysfunction.pptxJibrilAliSe
 
Erectile dysfunction
Erectile dysfunctionErectile dysfunction
Erectile dysfunctionPradeep Deb
 
Erectile Dysfunction Treatment Information by Premier Men's Medical Center
Erectile Dysfunction Treatment Information by Premier Men's Medical CenterErectile Dysfunction Treatment Information by Premier Men's Medical Center
Erectile Dysfunction Treatment Information by Premier Men's Medical CenterPremier Men's Medical Center
 
Nutrition And Erectile Dysfunction Public
Nutrition And Erectile Dysfunction PublicNutrition And Erectile Dysfunction Public
Nutrition And Erectile Dysfunction PublicRosen Wellness, LLC
 
management of Erectile Dysfunction.pptx
management of Erectile  Dysfunction.pptxmanagement of Erectile  Dysfunction.pptx
management of Erectile Dysfunction.pptxHarunausman10
 
Erectile dysfunction - causes and diagnosis
Erectile dysfunction - causes and diagnosisErectile dysfunction - causes and diagnosis
Erectile dysfunction - causes and diagnosisAbhishekPandey1012
 
Male sexual dysfunction By Sayed Eleweedy
Male sexual dysfunction By Sayed EleweedyMale sexual dysfunction By Sayed Eleweedy
Male sexual dysfunction By Sayed EleweedySayed Eleweedy
 
MA Erectile dysfunction.pptx
MA Erectile dysfunction.pptxMA Erectile dysfunction.pptx
MA Erectile dysfunction.pptxadamamadziga2
 
Erectile dysfunction updates
Erectile dysfunction updatesErectile dysfunction updates
Erectile dysfunction updatesMohamed Elgendy
 
Erectile dysfunction etiology presentation and diagnosis.pptx
Erectile dysfunction etiology presentation and diagnosis.pptxErectile dysfunction etiology presentation and diagnosis.pptx
Erectile dysfunction etiology presentation and diagnosis.pptxSonuKumarPlash
 
Erectile Dysfunction.pptx
Erectile Dysfunction.pptxErectile Dysfunction.pptx
Erectile Dysfunction.pptxSoumen Karmakar
 
Male reproductive system and its regulation
Male reproductive system and its regulationMale reproductive system and its regulation
Male reproductive system and its regulationMinko Syd
 
pe-15062106013e-150621060135-lva1-app6892.ppt
pe-15062106013e-150621060135-lva1-app6892.pptpe-15062106013e-150621060135-lva1-app6892.ppt
pe-15062106013e-150621060135-lva1-app6892.pptjavlon98madaminov
 
SCI_associated_conditions_Neurogenic_bladder_bowel_sexual_and.pptx
SCI_associated_conditions_Neurogenic_bladder_bowel_sexual_and.pptxSCI_associated_conditions_Neurogenic_bladder_bowel_sexual_and.pptx
SCI_associated_conditions_Neurogenic_bladder_bowel_sexual_and.pptxEgaRahman
 
Urinary incontinence in elderly
Urinary incontinence in elderlyUrinary incontinence in elderly
Urinary incontinence in elderlyRenu Kumawat
 

Ähnlich wie Erectile Dysfunction [Dr. Edmond Wong] (20)

4. Erectile dysfunction.pptx
4. Erectile dysfunction.pptx4. Erectile dysfunction.pptx
4. Erectile dysfunction.pptx
 
Erectile dysfunction
Erectile dysfunctionErectile dysfunction
Erectile dysfunction
 
Erectile Dysfunction Treatment Information by Premier Men's Medical Center
Erectile Dysfunction Treatment Information by Premier Men's Medical CenterErectile Dysfunction Treatment Information by Premier Men's Medical Center
Erectile Dysfunction Treatment Information by Premier Men's Medical Center
 
Nutrition And Erectile Dysfunction Public
Nutrition And Erectile Dysfunction PublicNutrition And Erectile Dysfunction Public
Nutrition And Erectile Dysfunction Public
 
management of Erectile Dysfunction.pptx
management of Erectile  Dysfunction.pptxmanagement of Erectile  Dysfunction.pptx
management of Erectile Dysfunction.pptx
 
Erectile dysfunction - causes and diagnosis
Erectile dysfunction - causes and diagnosisErectile dysfunction - causes and diagnosis
Erectile dysfunction - causes and diagnosis
 
Impotence
ImpotenceImpotence
Impotence
 
Erectile Dysfunction and Scleroderma: Evaluation and Managament
Erectile Dysfunction and Scleroderma: Evaluation and ManagamentErectile Dysfunction and Scleroderma: Evaluation and Managament
Erectile Dysfunction and Scleroderma: Evaluation and Managament
 
Male sexual dysfunction
Male sexual dysfunctionMale sexual dysfunction
Male sexual dysfunction
 
Male sexual dysfunction By Sayed Eleweedy
Male sexual dysfunction By Sayed EleweedyMale sexual dysfunction By Sayed Eleweedy
Male sexual dysfunction By Sayed Eleweedy
 
MA Erectile dysfunction.pptx
MA Erectile dysfunction.pptxMA Erectile dysfunction.pptx
MA Erectile dysfunction.pptx
 
Ejaculatory Disorders
Ejaculatory DisordersEjaculatory Disorders
Ejaculatory Disorders
 
Erectile dysfunction updates
Erectile dysfunction updatesErectile dysfunction updates
Erectile dysfunction updates
 
Erectile dysfunction etiology presentation and diagnosis.pptx
Erectile dysfunction etiology presentation and diagnosis.pptxErectile dysfunction etiology presentation and diagnosis.pptx
Erectile dysfunction etiology presentation and diagnosis.pptx
 
Erectile Dysfunction.pptx
Erectile Dysfunction.pptxErectile Dysfunction.pptx
Erectile Dysfunction.pptx
 
Andrology M.hassan & M.A.Wadood
Andrology  M.hassan & M.A.WadoodAndrology  M.hassan & M.A.Wadood
Andrology M.hassan & M.A.Wadood
 
Male reproductive system and its regulation
Male reproductive system and its regulationMale reproductive system and its regulation
Male reproductive system and its regulation
 
pe-15062106013e-150621060135-lva1-app6892.ppt
pe-15062106013e-150621060135-lva1-app6892.pptpe-15062106013e-150621060135-lva1-app6892.ppt
pe-15062106013e-150621060135-lva1-app6892.ppt
 
SCI_associated_conditions_Neurogenic_bladder_bowel_sexual_and.pptx
SCI_associated_conditions_Neurogenic_bladder_bowel_sexual_and.pptxSCI_associated_conditions_Neurogenic_bladder_bowel_sexual_and.pptx
SCI_associated_conditions_Neurogenic_bladder_bowel_sexual_and.pptx
 
Urinary incontinence in elderly
Urinary incontinence in elderlyUrinary incontinence in elderly
Urinary incontinence in elderly
 

Mehr von Edmond Wong

Infertility [Dr. Edmond Wong]
Infertility [Dr. Edmond Wong]Infertility [Dr. Edmond Wong]
Infertility [Dr. Edmond Wong]Edmond Wong
 
Human Renal Transplantation [Dr. Edmond Wong]
Human Renal Transplantation [Dr. Edmond Wong]Human Renal Transplantation [Dr. Edmond Wong]
Human Renal Transplantation [Dr. Edmond Wong]Edmond Wong
 
Emergency in Urology [Dr. Edmond Wong]
Emergency in Urology [Dr. Edmond Wong]Emergency in Urology [Dr. Edmond Wong]
Emergency in Urology [Dr. Edmond Wong]Edmond Wong
 
Neurogenic bladder [Dr. Edmond Wong]
Neurogenic bladder [Dr. Edmond Wong]Neurogenic bladder [Dr. Edmond Wong]
Neurogenic bladder [Dr. Edmond Wong]Edmond Wong
 
Muscle invasive bladder Cancer [Dr.Edmond Wong]
Muscle invasive bladder Cancer [Dr.Edmond Wong]Muscle invasive bladder Cancer [Dr.Edmond Wong]
Muscle invasive bladder Cancer [Dr.Edmond Wong]Edmond Wong
 
Incontinence & Female Urology [Dr.Edmond Wong]
Incontinence & Female Urology [Dr.Edmond Wong]Incontinence & Female Urology [Dr.Edmond Wong]
Incontinence & Female Urology [Dr.Edmond Wong]Edmond Wong
 
Urinary Stone Management [Dr. Edmond Wong]
Urinary Stone Management [Dr. Edmond Wong]Urinary Stone Management [Dr. Edmond Wong]
Urinary Stone Management [Dr. Edmond Wong]Edmond Wong
 
Paediatric Urology [Dr.Edmond Wong]
Paediatric Urology [Dr.Edmond Wong]Paediatric Urology [Dr.Edmond Wong]
Paediatric Urology [Dr.Edmond Wong]Edmond Wong
 
Bladder Cancer NMIBC [Dr.Edmond Wong]
Bladder Cancer NMIBC [Dr.Edmond Wong]Bladder Cancer NMIBC [Dr.Edmond Wong]
Bladder Cancer NMIBC [Dr.Edmond Wong]Edmond Wong
 
Urology New Technology and Imaging [Dr.Edmond Wong]
Urology New Technology and Imaging [Dr.Edmond Wong]Urology New Technology and Imaging [Dr.Edmond Wong]
Urology New Technology and Imaging [Dr.Edmond Wong]Edmond Wong
 
Urology infection [Dr. Edmond Wong]
Urology infection [Dr. Edmond Wong]Urology infection [Dr. Edmond Wong]
Urology infection [Dr. Edmond Wong]Edmond Wong
 
Urinary Diversion after cystectomy [Dr.Edmond Wong]
Urinary Diversion after cystectomy  [Dr.Edmond Wong]Urinary Diversion after cystectomy  [Dr.Edmond Wong]
Urinary Diversion after cystectomy [Dr.Edmond Wong]Edmond Wong
 
Upper Tract Transitional Cell Carcinoma [Dr. Edmond Wong]
Upper Tract Transitional Cell Carcinoma [Dr. Edmond Wong]Upper Tract Transitional Cell Carcinoma [Dr. Edmond Wong]
Upper Tract Transitional Cell Carcinoma [Dr. Edmond Wong]Edmond Wong
 
Testicular ca [edmond]
Testicular ca [edmond]Testicular ca [edmond]
Testicular ca [edmond]Edmond Wong
 
Ca penis [edmond]
Ca penis [edmond]Ca penis [edmond]
Ca penis [edmond]Edmond Wong
 
Ca kidney [edmond]
Ca kidney [edmond]Ca kidney [edmond]
Ca kidney [edmond]Edmond Wong
 
Ca prostate [edmond]
Ca prostate [edmond]Ca prostate [edmond]
Ca prostate [edmond]Edmond Wong
 

Mehr von Edmond Wong (17)

Infertility [Dr. Edmond Wong]
Infertility [Dr. Edmond Wong]Infertility [Dr. Edmond Wong]
Infertility [Dr. Edmond Wong]
 
Human Renal Transplantation [Dr. Edmond Wong]
Human Renal Transplantation [Dr. Edmond Wong]Human Renal Transplantation [Dr. Edmond Wong]
Human Renal Transplantation [Dr. Edmond Wong]
 
Emergency in Urology [Dr. Edmond Wong]
Emergency in Urology [Dr. Edmond Wong]Emergency in Urology [Dr. Edmond Wong]
Emergency in Urology [Dr. Edmond Wong]
 
Neurogenic bladder [Dr. Edmond Wong]
Neurogenic bladder [Dr. Edmond Wong]Neurogenic bladder [Dr. Edmond Wong]
Neurogenic bladder [Dr. Edmond Wong]
 
Muscle invasive bladder Cancer [Dr.Edmond Wong]
Muscle invasive bladder Cancer [Dr.Edmond Wong]Muscle invasive bladder Cancer [Dr.Edmond Wong]
Muscle invasive bladder Cancer [Dr.Edmond Wong]
 
Incontinence & Female Urology [Dr.Edmond Wong]
Incontinence & Female Urology [Dr.Edmond Wong]Incontinence & Female Urology [Dr.Edmond Wong]
Incontinence & Female Urology [Dr.Edmond Wong]
 
Urinary Stone Management [Dr. Edmond Wong]
Urinary Stone Management [Dr. Edmond Wong]Urinary Stone Management [Dr. Edmond Wong]
Urinary Stone Management [Dr. Edmond Wong]
 
Paediatric Urology [Dr.Edmond Wong]
Paediatric Urology [Dr.Edmond Wong]Paediatric Urology [Dr.Edmond Wong]
Paediatric Urology [Dr.Edmond Wong]
 
Bladder Cancer NMIBC [Dr.Edmond Wong]
Bladder Cancer NMIBC [Dr.Edmond Wong]Bladder Cancer NMIBC [Dr.Edmond Wong]
Bladder Cancer NMIBC [Dr.Edmond Wong]
 
Urology New Technology and Imaging [Dr.Edmond Wong]
Urology New Technology and Imaging [Dr.Edmond Wong]Urology New Technology and Imaging [Dr.Edmond Wong]
Urology New Technology and Imaging [Dr.Edmond Wong]
 
Urology infection [Dr. Edmond Wong]
Urology infection [Dr. Edmond Wong]Urology infection [Dr. Edmond Wong]
Urology infection [Dr. Edmond Wong]
 
Urinary Diversion after cystectomy [Dr.Edmond Wong]
Urinary Diversion after cystectomy  [Dr.Edmond Wong]Urinary Diversion after cystectomy  [Dr.Edmond Wong]
Urinary Diversion after cystectomy [Dr.Edmond Wong]
 
Upper Tract Transitional Cell Carcinoma [Dr. Edmond Wong]
Upper Tract Transitional Cell Carcinoma [Dr. Edmond Wong]Upper Tract Transitional Cell Carcinoma [Dr. Edmond Wong]
Upper Tract Transitional Cell Carcinoma [Dr. Edmond Wong]
 
Testicular ca [edmond]
Testicular ca [edmond]Testicular ca [edmond]
Testicular ca [edmond]
 
Ca penis [edmond]
Ca penis [edmond]Ca penis [edmond]
Ca penis [edmond]
 
Ca kidney [edmond]
Ca kidney [edmond]Ca kidney [edmond]
Ca kidney [edmond]
 
Ca prostate [edmond]
Ca prostate [edmond]Ca prostate [edmond]
Ca prostate [edmond]
 

Kürzlich hochgeladen

Call Girls Service Chennai Jiya 7001305949 Independent Escort Service Chennai
Call Girls Service Chennai Jiya 7001305949 Independent Escort Service ChennaiCall Girls Service Chennai Jiya 7001305949 Independent Escort Service Chennai
Call Girls Service Chennai Jiya 7001305949 Independent Escort Service ChennaiNehru place Escorts
 
Housewife Call Girls Bangalore - Call 7001305949 Rs-3500 with A/C Room Cash o...
Housewife Call Girls Bangalore - Call 7001305949 Rs-3500 with A/C Room Cash o...Housewife Call Girls Bangalore - Call 7001305949 Rs-3500 with A/C Room Cash o...
Housewife Call Girls Bangalore - Call 7001305949 Rs-3500 with A/C Room Cash o...narwatsonia7
 
Low Rate Call Girls Pune Esha 9907093804 Short 1500 Night 6000 Best call girl...
Low Rate Call Girls Pune Esha 9907093804 Short 1500 Night 6000 Best call girl...Low Rate Call Girls Pune Esha 9907093804 Short 1500 Night 6000 Best call girl...
Low Rate Call Girls Pune Esha 9907093804 Short 1500 Night 6000 Best call girl...Miss joya
 
See the 2,456 pharmacies on the National E-Pharmacy Platform
See the 2,456 pharmacies on the National E-Pharmacy PlatformSee the 2,456 pharmacies on the National E-Pharmacy Platform
See the 2,456 pharmacies on the National E-Pharmacy PlatformKweku Zurek
 
Kolkata Call Girls Services 9907093804 @24x7 High Class Babes Here Call Now
Kolkata Call Girls Services 9907093804 @24x7 High Class Babes Here Call NowKolkata Call Girls Services 9907093804 @24x7 High Class Babes Here Call Now
Kolkata Call Girls Services 9907093804 @24x7 High Class Babes Here Call NowNehru place Escorts
 
Call Girls Thane Just Call 9910780858 Get High Class Call Girls Service
Call Girls Thane Just Call 9910780858 Get High Class Call Girls ServiceCall Girls Thane Just Call 9910780858 Get High Class Call Girls Service
Call Girls Thane Just Call 9910780858 Get High Class Call Girls Servicesonalikaur4
 
Call Girls Service In Shyam Nagar Whatsapp 8445551418 Independent Escort Service
Call Girls Service In Shyam Nagar Whatsapp 8445551418 Independent Escort ServiceCall Girls Service In Shyam Nagar Whatsapp 8445551418 Independent Escort Service
Call Girls Service In Shyam Nagar Whatsapp 8445551418 Independent Escort Serviceparulsinha
 
Russian Call Girl Brookfield - 7001305949 Escorts Service 50% Off with Cash O...
Russian Call Girl Brookfield - 7001305949 Escorts Service 50% Off with Cash O...Russian Call Girl Brookfield - 7001305949 Escorts Service 50% Off with Cash O...
Russian Call Girl Brookfield - 7001305949 Escorts Service 50% Off with Cash O...narwatsonia7
 
Housewife Call Girls Hoskote | 7001305949 At Low Cost Cash Payment Booking
Housewife Call Girls Hoskote | 7001305949 At Low Cost Cash Payment BookingHousewife Call Girls Hoskote | 7001305949 At Low Cost Cash Payment Booking
Housewife Call Girls Hoskote | 7001305949 At Low Cost Cash Payment Bookingnarwatsonia7
 
Ahmedabad Call Girls CG Road 🔝9907093804 Short 1500 💋 Night 6000
Ahmedabad Call Girls CG Road 🔝9907093804  Short 1500  💋 Night 6000Ahmedabad Call Girls CG Road 🔝9907093804  Short 1500  💋 Night 6000
Ahmedabad Call Girls CG Road 🔝9907093804 Short 1500 💋 Night 6000aliya bhat
 
Call Girls Hsr Layout Just Call 7001305949 Top Class Call Girl Service Available
Call Girls Hsr Layout Just Call 7001305949 Top Class Call Girl Service AvailableCall Girls Hsr Layout Just Call 7001305949 Top Class Call Girl Service Available
Call Girls Hsr Layout Just Call 7001305949 Top Class Call Girl Service Availablenarwatsonia7
 
Call Girl Lucknow Mallika 7001305949 Independent Escort Service Lucknow
Call Girl Lucknow Mallika 7001305949 Independent Escort Service LucknowCall Girl Lucknow Mallika 7001305949 Independent Escort Service Lucknow
Call Girl Lucknow Mallika 7001305949 Independent Escort Service Lucknownarwatsonia7
 
call girls in green park DELHI 🔝 >༒9540349809 🔝 genuine Escort Service 🔝✔️✔️
call girls in green park  DELHI 🔝 >༒9540349809 🔝 genuine Escort Service 🔝✔️✔️call girls in green park  DELHI 🔝 >༒9540349809 🔝 genuine Escort Service 🔝✔️✔️
call girls in green park DELHI 🔝 >༒9540349809 🔝 genuine Escort Service 🔝✔️✔️saminamagar
 
Asthma Review - GINA guidelines summary 2024
Asthma Review - GINA guidelines summary 2024Asthma Review - GINA guidelines summary 2024
Asthma Review - GINA guidelines summary 2024Gabriel Guevara MD
 
Call Girls Electronic City Just Call 7001305949 Top Class Call Girl Service A...
Call Girls Electronic City Just Call 7001305949 Top Class Call Girl Service A...Call Girls Electronic City Just Call 7001305949 Top Class Call Girl Service A...
Call Girls Electronic City Just Call 7001305949 Top Class Call Girl Service A...narwatsonia7
 
Glomerular Filtration and determinants of glomerular filtration .pptx
Glomerular Filtration and  determinants of glomerular filtration .pptxGlomerular Filtration and  determinants of glomerular filtration .pptx
Glomerular Filtration and determinants of glomerular filtration .pptxDr.Nusrat Tariq
 
Call Girl Service Bidadi - For 7001305949 Cheap & Best with original Photos
Call Girl Service Bidadi - For 7001305949 Cheap & Best with original PhotosCall Girl Service Bidadi - For 7001305949 Cheap & Best with original Photos
Call Girl Service Bidadi - For 7001305949 Cheap & Best with original Photosnarwatsonia7
 
Hemostasis Physiology and Clinical correlations by Dr Faiza.pdf
Hemostasis Physiology and Clinical correlations by Dr Faiza.pdfHemostasis Physiology and Clinical correlations by Dr Faiza.pdf
Hemostasis Physiology and Clinical correlations by Dr Faiza.pdfMedicoseAcademics
 
Call Girls Frazer Town Just Call 7001305949 Top Class Call Girl Service Avail...
Call Girls Frazer Town Just Call 7001305949 Top Class Call Girl Service Avail...Call Girls Frazer Town Just Call 7001305949 Top Class Call Girl Service Avail...
Call Girls Frazer Town Just Call 7001305949 Top Class Call Girl Service Avail...narwatsonia7
 

Kürzlich hochgeladen (20)

Call Girls Service Chennai Jiya 7001305949 Independent Escort Service Chennai
Call Girls Service Chennai Jiya 7001305949 Independent Escort Service ChennaiCall Girls Service Chennai Jiya 7001305949 Independent Escort Service Chennai
Call Girls Service Chennai Jiya 7001305949 Independent Escort Service Chennai
 
sauth delhi call girls in Bhajanpura 🔝 9953056974 🔝 escort Service
sauth delhi call girls in Bhajanpura 🔝 9953056974 🔝 escort Servicesauth delhi call girls in Bhajanpura 🔝 9953056974 🔝 escort Service
sauth delhi call girls in Bhajanpura 🔝 9953056974 🔝 escort Service
 
Housewife Call Girls Bangalore - Call 7001305949 Rs-3500 with A/C Room Cash o...
Housewife Call Girls Bangalore - Call 7001305949 Rs-3500 with A/C Room Cash o...Housewife Call Girls Bangalore - Call 7001305949 Rs-3500 with A/C Room Cash o...
Housewife Call Girls Bangalore - Call 7001305949 Rs-3500 with A/C Room Cash o...
 
Low Rate Call Girls Pune Esha 9907093804 Short 1500 Night 6000 Best call girl...
Low Rate Call Girls Pune Esha 9907093804 Short 1500 Night 6000 Best call girl...Low Rate Call Girls Pune Esha 9907093804 Short 1500 Night 6000 Best call girl...
Low Rate Call Girls Pune Esha 9907093804 Short 1500 Night 6000 Best call girl...
 
See the 2,456 pharmacies on the National E-Pharmacy Platform
See the 2,456 pharmacies on the National E-Pharmacy PlatformSee the 2,456 pharmacies on the National E-Pharmacy Platform
See the 2,456 pharmacies on the National E-Pharmacy Platform
 
Kolkata Call Girls Services 9907093804 @24x7 High Class Babes Here Call Now
Kolkata Call Girls Services 9907093804 @24x7 High Class Babes Here Call NowKolkata Call Girls Services 9907093804 @24x7 High Class Babes Here Call Now
Kolkata Call Girls Services 9907093804 @24x7 High Class Babes Here Call Now
 
Call Girls Thane Just Call 9910780858 Get High Class Call Girls Service
Call Girls Thane Just Call 9910780858 Get High Class Call Girls ServiceCall Girls Thane Just Call 9910780858 Get High Class Call Girls Service
Call Girls Thane Just Call 9910780858 Get High Class Call Girls Service
 
Call Girls Service In Shyam Nagar Whatsapp 8445551418 Independent Escort Service
Call Girls Service In Shyam Nagar Whatsapp 8445551418 Independent Escort ServiceCall Girls Service In Shyam Nagar Whatsapp 8445551418 Independent Escort Service
Call Girls Service In Shyam Nagar Whatsapp 8445551418 Independent Escort Service
 
Russian Call Girl Brookfield - 7001305949 Escorts Service 50% Off with Cash O...
Russian Call Girl Brookfield - 7001305949 Escorts Service 50% Off with Cash O...Russian Call Girl Brookfield - 7001305949 Escorts Service 50% Off with Cash O...
Russian Call Girl Brookfield - 7001305949 Escorts Service 50% Off with Cash O...
 
Housewife Call Girls Hoskote | 7001305949 At Low Cost Cash Payment Booking
Housewife Call Girls Hoskote | 7001305949 At Low Cost Cash Payment BookingHousewife Call Girls Hoskote | 7001305949 At Low Cost Cash Payment Booking
Housewife Call Girls Hoskote | 7001305949 At Low Cost Cash Payment Booking
 
Ahmedabad Call Girls CG Road 🔝9907093804 Short 1500 💋 Night 6000
Ahmedabad Call Girls CG Road 🔝9907093804  Short 1500  💋 Night 6000Ahmedabad Call Girls CG Road 🔝9907093804  Short 1500  💋 Night 6000
Ahmedabad Call Girls CG Road 🔝9907093804 Short 1500 💋 Night 6000
 
Call Girls Hsr Layout Just Call 7001305949 Top Class Call Girl Service Available
Call Girls Hsr Layout Just Call 7001305949 Top Class Call Girl Service AvailableCall Girls Hsr Layout Just Call 7001305949 Top Class Call Girl Service Available
Call Girls Hsr Layout Just Call 7001305949 Top Class Call Girl Service Available
 
Call Girl Lucknow Mallika 7001305949 Independent Escort Service Lucknow
Call Girl Lucknow Mallika 7001305949 Independent Escort Service LucknowCall Girl Lucknow Mallika 7001305949 Independent Escort Service Lucknow
Call Girl Lucknow Mallika 7001305949 Independent Escort Service Lucknow
 
call girls in green park DELHI 🔝 >༒9540349809 🔝 genuine Escort Service 🔝✔️✔️
call girls in green park  DELHI 🔝 >༒9540349809 🔝 genuine Escort Service 🔝✔️✔️call girls in green park  DELHI 🔝 >༒9540349809 🔝 genuine Escort Service 🔝✔️✔️
call girls in green park DELHI 🔝 >༒9540349809 🔝 genuine Escort Service 🔝✔️✔️
 
Asthma Review - GINA guidelines summary 2024
Asthma Review - GINA guidelines summary 2024Asthma Review - GINA guidelines summary 2024
Asthma Review - GINA guidelines summary 2024
 
Call Girls Electronic City Just Call 7001305949 Top Class Call Girl Service A...
Call Girls Electronic City Just Call 7001305949 Top Class Call Girl Service A...Call Girls Electronic City Just Call 7001305949 Top Class Call Girl Service A...
Call Girls Electronic City Just Call 7001305949 Top Class Call Girl Service A...
 
Glomerular Filtration and determinants of glomerular filtration .pptx
Glomerular Filtration and  determinants of glomerular filtration .pptxGlomerular Filtration and  determinants of glomerular filtration .pptx
Glomerular Filtration and determinants of glomerular filtration .pptx
 
Call Girl Service Bidadi - For 7001305949 Cheap & Best with original Photos
Call Girl Service Bidadi - For 7001305949 Cheap & Best with original PhotosCall Girl Service Bidadi - For 7001305949 Cheap & Best with original Photos
Call Girl Service Bidadi - For 7001305949 Cheap & Best with original Photos
 
Hemostasis Physiology and Clinical correlations by Dr Faiza.pdf
Hemostasis Physiology and Clinical correlations by Dr Faiza.pdfHemostasis Physiology and Clinical correlations by Dr Faiza.pdf
Hemostasis Physiology and Clinical correlations by Dr Faiza.pdf
 
Call Girls Frazer Town Just Call 7001305949 Top Class Call Girl Service Avail...
Call Girls Frazer Town Just Call 7001305949 Top Class Call Girl Service Avail...Call Girls Frazer Town Just Call 7001305949 Top Class Call Girl Service Avail...
Call Girls Frazer Town Just Call 7001305949 Top Class Call Girl Service Avail...
 

Erectile Dysfunction [Dr. Edmond Wong]

  • 2. A B C • Name the structures A, B, C and D (0.5 mark each) D
  • 3. • A : Skin (0.5) • B : Dartos fascia (0.5) • C : Buck’s fascia (0.5) • D : Tunica albuginea (0.5)
  • 4. What is the definition of ED? • Persistent inability to initiate and maintain an erection sufficient for satisfactory sexual activity • Better not to use “Impotence” as it is less precisely defined
  • 5. What is prevalence & severity? • Massachusetts Male Aging Study (MMAS) • Prevalence – – Men 40-70yo • ~50% have ED • Mild ED: 17% • Moderate ED : 25% • Complete ED : 10% • ED prevalence increases with age – 50% at age 50, 60% at age 60, 70% at age 70
  • 6. What is innervation of erection and ejaculation? • Autonomic – Sympathetic nerves from T11-L2 – Parasympathetic from S2-4, form the pelvic plexus – The cavernosal nerves are branches of pelvic plexus (i.e. parasympathetic) that innervate the penis – Parasympathetic stimulation causes erection – Sympathetic activity causes ejaculation and detumescence (loss of erection)
  • 7. What is innervation of erection and ejaculation? • Somatic – Somatosensory information travels via the pudendal nerves – Onuf’s nucleus (S2-4) is the somatic centre for efferent innervation of the ischiocavernosus and bulbocavernosus muscles of the penis • Central – Medial preoptic area and paraventricular nucleus (PVN) in the hypothalamus are important centres for sexual function and penile erection
  • 8. What is the arterial blood supply of penis? • Originate from internal pudendal artery • Three branches of common penile artery which join to form vascular ring near the glans • Bulbourethral artery – Supplying the bulb and corpus spongiosum • Dorsa penile artery – Skin, fascia and the glans penis and forms anastomoses with the bulbourethral artery at he glans. These anastomoses allow division the the urethra during urethral stricture surgery without compromising the blood supply to the distal urethral • The cavernous artery – Supplies only the cavernosal bodies & gives off many helicine arteries which supply the trabecular erectile tissue & sinusoid. Does not anastomose with other 2 penile arteries • Pulsations absent in penile vessels – likely vascular cause of ED
  • 9. What is the venous blood supply of penis? • Skin and subcutaneous tissue  saphenous vein • Emissary veins draining corpus cavernosum and corpus spongiosum  deep dorsal vein  periprostatic venous plexus • Emissary veins draining proximal corpora cavernosa  nternal pudendal veins
  • 10. Physiology of Erectile ResponsePhysiology of Erectile Response Complex process combining • psychological stimuli • neurologic event • smooth muscle relaxation • arterial dilation • venous compression
  • 11. What are the five phases of erection? Phase Term 0 Flaccid phase 1 Latent (filling) phase 2 Tumescent phase 3 Full erection phase 4 Rigid erection phase 5 Detumescence phase
  • 12. What is mechanism of erection? • Neuroendocrine signals from the brain, created by audiovisual or tactile stimuli • Signals are relayed via the cavernosal nerve to the erectile tissue of the copora cavernosa, activating the veno-occlusive mechanism • This triggers increased arterial blood flow into sinusoidal spaces with relaxation of cavernosal smooth muscle, and opening of the vascular space • Compressing the subtunical venous plexuses, decreasing venous outflow • Both spongiosus and cavernosus are surrounded by tunica albuginea, which consist of outer longitudinal and inner circular layers. The sliding of 2 layers over each other during engorgement lead to occlusion of emissary veins • Rising intracavernosal pressure and contraction of the ischiocavernosus muscles produces a rigid erection
  • 13. Mechanism of erection • Full erection phase: – Compression of the deep dorsal and cricumflex vein btw Buck’s fascia & engorged cavernosa  glanular tumescence • Rigid –erection phase: – Ischiocavernosus and bulbocavernosus muscle forcefully compress the spongiosum and penile veins  further engorgement and increase pressure in glans and spongiosum
  • 14. Mechanism of PDE5-i • nitric oxide is synthesized from L-arginine and released by neurons, endothelial cells, and possibly corporal smooth muscle cells of the penis in response to sexual stimulation • Nitric oxide enter SM cell • Activate soluble form of enzyme guanylate cyclase (sGC) • sGC convert guanosine triphosphate (GTP) to cyclic guanosine monophosphate (cGMP) (an active intracellular 2nd messenger) • cGMP lead to SM relaxation thru reduction of intracellular Ca  erection • cGMP is metabolize to GMP (inactive) by PDEs • Thus PDE inhibitor facilitate NO induced SM relaxation by increase accumulation of intracellular cGMP
  • 15.
  • 16.
  • 17. What is mechanism of erection?
  • 18. Pathophysiology of Erectile Dysfunction Inflow Outflow Failure to initiate Psychogenic / Neurogenic Failure to store Venous leak Failure to fill Arterial insufficiency
  • 19. What is the mechanism of venogenic erectile dysfunction?
  • 20. • What physiological process is this chart describing? (1) • What are the top and bottom row depicting? (2) • Can you name the 6 columns? (2) Q57
  • 21. • Tumescence / erection (1) • Top row : artist depiction of the state of the cavernosal arteries during different phases of erection induced by prostaglandin injection (1) • Bottom row : Doppler waveform of the cavernous arteries during this erection (1) • Flaccid, latent, tumescent, full, rigid, detumescent • (2 marks for ALL 6 correct answers)
  • 22. What is the mechanism of ejaculation? • Tactile stimulation of the glans penis causes sensory information to travel (via the pudendal nerve) to the lumbar spinal sympathetic nuclei • Sympathetic efferent signals (travelling in the hypogastric nerve) cause contraction of smooth muscle of the epididymis, vas deferens, and secretory glands, propelling spermatozoa and glandular secretions into the prostatic urethra • There is simultaneous closure of the internal urethral sphincter and relaxation of the extrinsic sphincter • Rhythmic contraction of the bulbocavernosus muscle leads to the pulsatile emission of the ejaculate from the urethra
  • 23. Risk Factors for ED • Aging • Systemic diseases: D.M., hypertension, atherosclerosis, hyperlipidemia • Endocrine disorders: hypogonadism, hyperprotactinemia • Alcohol abuse or smoking • Trauma or surgery to pelvis or spine • Depression or stress • Drugs: antihypertensives, antidepressants, hormones
  • 24. What are the causes of ED? • IMPOTENCE – Inflammatory – prostatitis – Mechanical – Peyronie’s disease – Psychological – depression – Occlusive vascular factors – arteriogenic, PVD – Trauma – pelvic fracture – Extra factors – pelvic surgery/RRP – neuro (cavernosal nerve)-vascular (to cavernosa) injury – Neurogenic – stroke/spinal cord defect – Chemical – alcohol, smoking, drugs (diuretics/anti-HT) – Endocrine – DM
  • 25. Aetiological Causes Psychogenic • anxiety • depression Endocrine • hormonal deficiency Neurogenic • surgery or trauma to pelvis or spine • diabetes mellitus or alcohol
  • 26. Aetiological Causes Arteriogenic • hypertension, smoking, D.M. • hyperlipidaemia • surgery or trauma to pelvis Venous • functional impairment of the veno-occlusive mechanism Drugs • antihypertensives, antidepressants
  • 27. What medication asso with ED? • Diuretics & antihypertensive • Antidepressant • Anti-anxiety • Anti-epileptic drugs • Pakinson’s disease med • Antihistamines • NSAIDs • Antiarrhythmics • H-2 blockers • Muscle relaxants • Prostate cancer medication • Chemotherapy
  • 28. What is the difference between psychogenic vs organic causes? • Psychogenic – Sudden onset – Situational – Morning erection +ve – Rigidity of morning erection – Other psychological complaints – Spouse relationship – Abnormal sexual development • Organic – Gradual onset – Always – Morning erection –ve – Chronic medical illness (DM, HT, IHD) – Pelvis trauma / surgery – Endocrine / Neurological disease – Recreational drugs – +/- Loss of libido – +/- Reduced size of penis
  • 30. Scenario • M/70 • HT/DM • Worsening erectile function for 1 year • Affecting relationship with 30-year-old wife
  • 31. What is the specific history for ED? • Sexual – Onset (sudden or gradual) – Duration, severity – IIEF – Early morning erections? – Loss of libido, sexual relationship issues (different partners) – Previous treatment – Patient’s expectation • Medical and surgical – Hypertension; cardiac disease; peripheral vascular disease; diabetes mellitus; endocrine or neurological disorders; pelvic surgery, radiotherapy, or trauma • Drugs – Antipsychotic drugs, anti-anxiety drugs, diuretics, chemotherapy, etc • Psychosocial – Anxiety, depression • Social – Smoking, alcohol consumption
  • 32. What is IIEF? • IIEF (International Index of Erectile Function) 1997 • For assessment of intensity of ED • Best description of own situation in last 6 months of sexual activity
  • 33. What is IIEF? • Five domains 1. Erectile function 2. Orgasmic function 3. Sexual drive 4. Intercourse satisfaction 5. Overall satisfaction • Short form: IIEF-5 – Scoring 1-5 for each, Total 25 – No ED: 22-25 – Mild ED: 17-22 – Mild to moderate: 12-16 – Moderate ED: 8-11 – Severe ED 1-7
  • 35. What is the physical examination? • Height , weight BMI , BP • Secondary sexual characteristics to rule out hypogonadism • Thyroid evaluation • Cardiovascular system – LL pulses • Abdominal – Waist cricumference • Neurological system – Penile Sensatiion, bulbocarvernosus reflex, LL neurology • Genital-urinary system – Penile deformity, phimosis, Peyronie's plaques – Testicular size, consistence and mass – DRE: anal tone, prostate
  • 36. What is the Ix? • L/RFT • PSA • Fasting glucose • Lipid profile • TFT • Hormone profile – Testosterone – Prolactin • Urinalysis
  • 37. Why should ED be investgiated ED is associated with other morbidities in 20% • DM • Occult cardiac disease • Dyslipidaemia • Endocrine disorders: hypogonadism • CVA – 50%
  • 39. What is cardiac evaluation in ED? • If happened to men having symptomatic coronary artery disease ( CAD ), ED precedes 55 – 65% CAD by 3 – 4 years • Sex energy expenditure is equal to 15min walk or climbing 2 flights • Absolute risk that sex can trigger MI is 1- 2 per million
  • 40. Princeton II consensusPrinceton II consensus ED as warning sign of vascular / cardiac disease
  • 41. What is Princeton II consensus?
  • 42. What is Princeton II consensus?
  • 43. What is Princeton II consensus? • Low risk – Controlled HT, NYHA 1, <3 risk factors for IHD, stable angina, uncomplicated past MI • Intermediate risk – >/= 3 risk factors, NYHA 2, <6 weeks from MI • High risk – High risk arrhythmia, unstable angina, recent MI (<2 weeks), HOCM, NYHA 3-4, uncontrolled HT
  • 44. What are the indications for specific diagnostic tests? • Primary erectile disorder (not caused by organic disease or psychogenic disorder) • Young patients with a history of pelvic or perineal trauma • Patients with penile deformities that might require surgical correction, e.g. Peyronie’s disease, congenital curvature • Those unresponsive to medical therapy • Complex psychiatric or psychosexual disorders • Complex endocrine disorders • Medicolegal reasons, e.g. implantation of penile prosthesis, sexual abuse
  • 45. What are the special Test? 1. Nocturnal penile tumescence (NPT) 2. Caverject Trial 3. Vascular imaging 4. Duplex USG 5. Cavernosometry 6. Penile arteriography
  • 46. What is nocturnal penile tumescence testing? • 80% Noctural Penile Tumescence (NPT) occurs during REM sleep • Rigiscan device contains 2 rings which are placed around base and distal penile shaft to measure tumescence and number, duration, and rigidity of nocturnal erections • Should be done on at least two nights. • A functional erectile mechanism is indicated by an erectile event of – at least 60% rigidity recorded on the tip of the penis – that lasts for 10 min or more • Gold Standard – Organics vs Psychogenic
  • 47. What is nocturnal penile tumescence testing?
  • 48. What is intracavernous injection test? • A positive test is a rigid erectile response (unable to bend the penis) that appears within 10 min after the intracavernous injection and lasts for 30 min • This response indicates a functional and rule out veno-occlusive dysfunction, although co-exist with arterial insufficiency • If inconclusive as a diagnostic procedure and Duplex ultrasound of the penile arteries should be requested • Assisted in Ix including Duplex USG / DICC / penile arteriogram • Positive test shows that a patient will respond to the intracavernous injection program
  • 49. What is duplex ultrasound of penile arteries? • Assess cavernosal arterial inflow to corpora cavernosa • Normal value – Peak systolic blood flow >35 cm/s – End diastolic velocity <5 cm/s and – Resistance index >0.8 • Further vascular investigation is unnecessary when a Duplex examination is normal
  • 50. • Name this investigation (1) • In investigating what condition is this used? (1) • What is the finding shown? (1) Q27
  • 51. • Pharmacologic cavernosography (1)/ Dynamic infusion cavernosgraphy & cavernosometry (DICC) (1) • ED (1) • Venous leakage along pelvic veins suggestive of veno-occlusive dysfunction (1)
  • 52. Cavernosography • Indication: 1. evaluate venous problems in men with ED 2. Investigation of priapism (high flow) 3. Assessment of penile fractures/injury to assess cavernosal damage 4. Assessment of Peyronie’s disease (rarely used) • Contraindication: – Hx of contrast allergy
  • 53. Carvernosography • Two 19–22 G butterfly needles inserted into the corpora • 60-100ml Omnipaque or urograffin infused slowly to obtain penile pressure 90mmHg • If penis not erection , contrast leakage • Fluoroscopy: AP , Rt, Lt oblique view • Normal: no contrast visualized outside the 2 corpora cavernosa • Abnormal: Contrast leakage or significant curvature • Patient asked to squeeze penis for 5min to ensure complete emptying
  • 54. • Advantage: more sensitive and accurate compare to doppler USG for venous leakage • Disadvantage: – Invasive – Can be painful – Risk of infection – Contrast related fibrosis within corpora – Risk of priapism
  • 55. Dynamic Infusion Cavernosometry & Cavernosography • 4 phases – Injection of vasoactive agents (alprostadil, Bimix, Trimix) into one corpus cavernosum to relax the corporeal smooth muscles – Pharmacologic cavernosometry (infusing the penis with heparinized saline whilst monitoring the intracavernosmal pressure) – Cavernosal artery systolic occlusion pressure (CASOP) reached as intracavernosal pressure drops – Pharmacologic cavernosography (infusing contrast into the corporeal tissue and obtaining radiographic images of the penis and perineum to see if there is venous leakage • Normal: – A gradient between the CASOP and the brachial artery pressures of <35mmHg – an equal pressure between the right and the left cavernous arteries Venous leakage: -Inability to occlude systolic pressure -Large gradient between CASOP & brachial systolic pressure -Rapid drop of intracavernosal pressure upon stopping of infusion
  • 56. What is the indication of arteriography and dynamic infusion cavernosometry or cavernosography? • Arteriography and DICC: dynamic infusion cavernosometry or cavernosography should be performed only in patients to rule out venous leakage who are being considered for vascular reconstructive surgery
  • 58. Treatment of Erectile Dysfunction • Treat underlying disease to preserve health • Elimination of modifiable risk factors • Disorders that need treatment as part of ED management - relationship conflict - depression, psychogenic ED - hypogonadism, hyperprolactinaemia
  • 59. Treatment of Erectile Dysfunction • Cardiovascular status of the patient • Is the patient able to resume sexual activity? - if not, cardiovascular assessment and intervention may be appropriate • Patient and partner choices play important role in identifying successful treatment
  • 60. What are the treatment options? • First-line therapies – Lifestyle modification, psychosexual therapy – PDE5i • Second-line therapies – Intraurethral injection of alprostadil – Intracavernosal injection of alprostadil – Vacuum constriction devices • Third-line therapy – Surgical implantation of prosthesis – Penile vascular reconstruction
  • 61. What is the lifestyle modification to improve ED? • Smoking • Alcohol • HT, DM • Obesity (BMI), exercise • A multicentre, randomised study – In obese men with moderate ED compared 2 years of intensive exercise and weight loss – Significant improvements in body mass index (BMI) and physical activity scores, as well as in erectile function • Esposito K, et al. Effect of lifestyle changes on erectile dysfunction in obese men: a randomized controlled trial. JAMA 2004 Jun;291(24):2978-84
  • 62. What is pyschosexual therapy? • Identify and treat underlying psychological diseases • Provides information and treatment in the form of sex education, instruction on improving partner communication skills, cognitive therapy, and behavioral therapy
  • 63. What is the general efficacy of various treatment? • PDE5i - 80% success – DM 60%, NSRRP 70% • ICI - 90% • VCD - 90% • MUSE - 50%
  • 65. What is Phosphodiesterase type-5 (PDE5) inhibitors? • sildenafil (Viagra) – half-life 4h • tadalafil (Cialis) – half-life 17.5h • vardenafil (Levitra) – half-life 4h • All have similar effect and outcome • Enhance cavernosal smooth muscle relaxation and erection by blocking the breakdown of cGMP to 5GMP. Sexual stimulus is still required to initiate events • Make sure no Contraindications – Concomitant use of nitrates – Hx of retinitis pigmentosa – Princeton high risk group – Severe liver function impairment • IC50 is the concentration of drug required to produce 50% inhibition of target enzyme. Vardenafil has the lowest IC50 0.7nM compared to 0.9 for tadalafil and 3.5 for sildenafil
  • 66. Character of PDE5-i Sildenafil (Viagra) Vardenafil (Levitrat) Tadalafil (Cialis) Onset of action 15 min -1hr 15min -1hr 15min -2 hr Effect of food Reduced absorption with fatty food Reduced absorption with fatty food NONE Dosage 20,50,100mg 5,10,20mg 5,10,20mg Side effects Headache, dyspepsia, facial flushing, blurred/blue vision, backache, myalygia Headache, dyspepsia, facial flushing, blurred/blue vision, backache, myalgia Prolong QT Headache, dyspepsia, facial flushing , blurred/ blue vision , backache, myalgia Contraindications Nitrates Nitrates, anti- arrhythmics Nitrates
  • 67. How to counsel patient for PDE5-i? • No data comparing the efficacy for sildenafil, tadalafil and vardenafil • Choice of drug will depend on the patient preference – Unplanned sex: tadalafil – Planned sex: Sildenafil & verdenafil • Taken at least 30min to 1hr before sex, with empty stomach • Medication need appropriate sexual stimulation • Explained potential side effect
  • 68. Adverse effect • Headache (20%) • Flushing (15%) • Dyspepsia (10%) • Rhinitis (6%) • Blurred/ blue vision (6%) (Chromatopsia) – Cross activity with PDE6 (retinal phototransduction enzyme) • Myalgia and back pain (5%) • Dizziness (5%) • Priapism (rare)
  • 69. What to do if failed PDE5-i • 20% do not respond to any PDE5 inhibitors • Can change to another if one ineffective • Exclude fake drug • Should at least try 4 times for at least 2 drug with maximal dosage before considering failure • Re-education: – Dose , timing of med – Alcohol, interaction with fatty food – Adequate sexual arousal – Try few more times • Check testosterone: make sure not hypogonadal • Addition testosterone if hypogonadal: – General improvement in sexual function – Improved erection – Enhanced responsiveness of PDE5-I • Lipitor (Atorvastatin) improve response to sildenafil (Hermann JSM 2005)
  • 70. What is non-arteritic anterior ischaemic optic neuropathy? • FDA alert 7/2005: small number of men had lost of eyesight after taking Levitra, viagra and cialis. This non arteritic ischemic optic neuropathy cause sudden painless loss of eyesight because blood flow to the optic nerve is blocked. (disc edema) • It is not known whether these drugs cause NAION, as the condition also occur in men not taking such drugs • High risk patients for NAION include – Over 50 – DM – Hypertension – High cholesterol – Smoker – Certain eye problem
  • 71. How about Cardiovascular safety? • No increase in myocardial infarction rates • Nitrates are totally contraindicated with PDE5 inhibitors – cGMP accumulation and unpredictable falls in blood pressure – If PDE5I is taken and the patient develops chest pain, nitroglycerine must be withheld for at least 24 hours for viagra and levitra, and for at least 48 hours for cialis
  • 72. What is the evidence of Changing the PDE5 inhibitor in non- responder? • A randomized, open-label, crossover trial comparing sildenafil and tadalafil • Some patients might respond better to one PDE5 inhibitor than to another and vice versa • Might be explained by variation in drug pharmacokinetics – Eardley I, et al. Factors associated with preference for sildenafil citrate and tadalafil for treating erectile dysfunction in men naive to phosphodiesterase 5 inhibitor therapy: post hoc analysis of data from a multicentre, randomized, open-label, crossover study. BJU Int 2007 Jul;100(1):122-9
  • 73. What is the evidence of regular dosing of PDE5 inhibitor in non-responder? • No randomized trials to support this intervention • Although tadalafil is licensed for daily dosing at a dose of 2.5 mg and 5 mg, neither sildenafil nor vardenafil are licensed for use in this way
  • 74. What is the difference between on- demand and chronic use of PDE5 inhibitors? • Double-blind, placebo-controlled, multicentre, parallel-group study • Mild-to-moderate ED randomised to receive once-daily vardenafil 10 mg plus on-demand vardenafil • Once-daily dosing does not offer any sustainable effect after cessation of treatment
  • 75. What is the evidence of PDE5 inhibitors in post-RRP? • Early use of a high dose of sildenafil after RP is associated with the preservation of smooth muscle within the human corpora cavernosa • The response rate to sildenafil treatment for ED after RP in different trials up to 70% among those who underwent bilateral NSRP and up to 15% among those who underwent non-NSRP • Daily sildenafil also resulted in a greater return of spontaneous normal erectile function post RP compared to placebo following bilateral nerve-sparing RP in patients who were fully potent before surgery
  • 76. What is the evidence of PDE5 inhibitors in post-RRP? • A randomized, double-blind, multicentre study • Compared on- demand and nightly dosing of vardenafil in men with ED following bilateral NSRP • Vardenafil was efficacious when used on demand, supporting a paradigm shift towards on-demand dosing Montorsi F, et al. Effect of nightly versus on-demand vardenafil on recovery of erectile function in men following bilateral nerve-sparing radical prostatectomy. Eur Urol 2008;54(4):924-31
  • 77. What is the precaution with anti-hypertensive drugs? • Small additive drops in blood pressure, which are usually minor • Even when the patient is taking several antihypertensive agents
  • 78. How about Alpha-blocker interactions? • Viagra: 50, 100mg not to be taken within a 4 hour window of an alpha blocker • Levitra: safe to use with tamsolusin. (label changed in US from previous total contraindication with alpha blockers) • Cialis: safe to use with tamsolusin 0.4mg
  • 80. What is Apomorphine? • Dopamine receptor agonist • Sublingual (Uprima SL) – Erections are achieved within 20 min • Efficacy rates (erections sufficient for intercourse) range from 30% to 55%) • Acts centrally on dopaminergic receptors in the paraventricular nucleus of the hypothalamus to enhance and co-ordinate the effect of sexual stimuli • Adverse effects: nausea; headache; dizziness • Apomorphine is not contraindicated in patients taking nitrates or antihypertensive drugs • Used in patients with certain contraindications for the use of PDE5 inhibitors, e.g. nitrates • Multiple daily dosing possible: once / 8 hours
  • 82. What is the mechanism of prostaglandin E? • Prostaglandin E binds to PGE receptor, activating adenylate cyclase which converts ATP to cAMP • cAMP activate PKA which brings about the relaxation of the smooth muscle directly without NO pathway
  • 83.
  • 84. What is intraurethral therapy? • Alprostadil • Synthetic prostaglandin E1 (PGE1) pellet administered into the urethra via a specialized applicator • Once inserted, the penis is gently rolled to encourage the pellet to dissolve into the urethral mucosa, from where it enters the corpora • Efficacy – 50% • Side-effects: penile pain; priapism; local reactions
  • 86. What is Intracavernosal therapy? • Alprostadil • Increase cAMP within corporal smooth muscle  relaxation of SMC • Right angle at lateral mid-penile shaft • Efficacy rates for intracavernous alprostadil - 90% • Adverse effects: pain; priapism; haematoma • Contraindications: men at risk of priapism (1%) and men with bleeding disorders
  • 88. What is Intracavernosal therapy? Locate the area of injection. Wipe off with an alcohol swab. Grasp the head of the penis, not the skin. Position the penis along your inner thigh. Maintain traction on the head after cleaning the side of the penis.
  • 89. What is Intracavernosal therapy? • Most long-term injection users can switch to sildenafil despite underlying pathophysiology • Almost one-third of long-term intracavernous injections users who subsequently responded also to sildenafil preferred to continue with an intracavernous injection programme
  • 90. What is combination therapy? • Papaverine – combination therapy today due to its high incidence of side-effects as monotherapy • Phentolamine - in combination therapy to increase efficacy. As monotherapy, it produces a poor erectile response
  • 91. What is combination therapy? • Papaverine plus phentolamine plus alprostadil have never been licensed for ED • Combination had similar side-effects as alprostadil monotherapy, but a lower incidence of penile pain due to lower doses of alprostadil • Fibrosis and priaprism were more common when papaverine was used. In addition, mild hepatotoxicity has been reported with papaverine
  • 92. What is the action to be taken with a prolonged erection? • 19-gauge needle is used to aspirate blood • If failed, an intracavernous injection of phenylephrine, starting at a dose of 200 μg every 5 min and increasing to 500 μg if necessary
  • 93. Vacuum erection deviceVacuum erection device
  • 95. • What is this device? (0.5) • Name one condition it is used for (0.5) • Name one complication from its use? (1)
  • 96. • Vaccum constriction device (0.5) • ED, Peyronie’s disease (0.5) • Cold penis, difficulty with ejaculation, bruising, penile numbness (1 max)
  • 97. Vaccum constriction device • Erections with these do not use physiological erection pathways • Plastic cylinder connected directly or by tubing to a vacuum- generating source (manual or battery-operated pump) • Penis is engorged by the negative pressure • Constricting ring is applied to the base to maintain the erection  should not be left in place > 30 minutes • Can be used successfully by men with a malfunctioning penile prosthesis in place • Used after explanation to prevent shortening • Disadvantage: cold numb penis, no ejaculation, discomfort in orgasm • Patients taking aspirin or warfarin should exercise caution when using these devices • Erections satisfactory for intercourse, is as high as 90%, but decreases to 50-64% after 2 years. Most men who discontinue within 3 months
  • 99.
  • 100. What is Penile prosthesis? • Third-line therapy • Surgical implantation into the corpora to provide penile rigidity and sufficient erectile size for sexual intercourse • Two types of prosthesis exist: malleable (semi-rigid) and inflatable (two- or three-piece) • Most patients prefer the three-piece inflatable devices due to the more ‘natural’ erections obtained. • Two-piece inflatable prosthesis can be a reliable option with fewer mechanical complications and is easier to implant • A semi-rigid prosthesis provides a constantly rigid penis and may be suitable in older patients with infrequent sexual intercourse with less mechanical failure but erosion and chronic pain
  • 101. • AMS 700 • 5YS 90% • Side-effects: mechanical failure (5% per year); erosions (5%); infections (2%) • Reservoir in abdomen • Pump in scrotum, pair of cylinders implant into penis • Infection rate may be reduced to 1% by implanting an antibiotic- impregnated prosthesis • Infection rate is similar between primary VS revision, DM VS non-DM • Staphylococcus epidermidis- commonest • Mulcahy technique has been described in an attempt to salvage the situation and insert a new prosthesis at the time of the infected one. It involves copious wound irrigation with kanamycin, bacitracin, iodine, hydrogen peroxide, vancomycin and gentamycin. The success rate is 80% at 3 years
  • 103. What is surgical penile revascularization? • Post-traumatic arteriogenic ED in young patients • In young patients with pelvic or perineal trauma, surgical penile revascularization has a 70% long- term success rate • The lesion must be demonstrated by Duplex ultrasound and confirmed by selective internal pudendal arteriogram • Corporeal veno-occlusive dysfunction is a contraindication to revascularization and must be excluded by DICC • Vascular surgery for veno-occlusive dysfunction is no longer recommended because of poor long- term results
  • 105. hypogonadism • 95% testosterone is produce by Leydig cells • Serum level peak at 7-8am and lowest at midnight. Clinically serum testosterone is best estimated in the morning • Incidence of late-onset hypogonadism: 20% of men over 70 • <10% ED due to hypogonadism • Serum testosterone is loosely bound to albumin in 50% and 50% to sex hormone binding globulin (SHBG). Whereas 2% is free. The bioactive testosterone refers to free and albumin bound serum testosterone (~50%) • Estimation of the bioavailable testosterone is a more accurate test than total serum testosterone when investigating hypogonadism • SHBG is increase in: ageing , cirrhosis, hyperthyroidism , anticovulsants, oestrogen, HIV infection
  • 106. Approach • History: – loss of libido, low mood, lethargy – Change in sexual function • Physical examination: – Loss of muscle mass & hair loss – regression of secondary sexual characteristics – softer and smaller testis, – gynaecomastia (aromatisation of testosterone in fatty tissue to oestrogen) • Investigation: – fasting glucose, lipid, – total testosterone, free testosterone, FSH and LH – Oestradiol, TFT & prolactin in selected case
  • 107. Late-onset male hypogonadism • Symptom complex resulting from age- related decline in testosterone level in men • Cause: – Primary: Testicular failure – Secondary: • Pituitary or hypothalamic disorder (Kallman’s syndrome) • Conbined hypogonadism
  • 108. Treatment • Weight loss • Clomiphene citrate • Androgen replacement therapy –CI: polycythemia, fluid retention, Ca prostate and breast , sleep apnoea, heart failure, severe renal and liver failure –Oral – first pass to liver for metabolism > too fast to have an effect –Transdermal, subcutaneous, intramuscular
  • 109. Monitor • Serum testosterone level • Clinical signs and symptoms • Assess bone mineral density • Adverse effect: – Excessive rise in hematocrit (>54%) – Raised PSA or abnormal DRE – Ance – Increase oliness of skin – Gynaecomastia – Suppression of fertility – Some testicular atrophy
  • 110. Testosterone & Ca Prostate? • No positive correlation btw testosterone level and Ca Prostate in prospective epidemiological studies • Small clinical trails have not shown increase in clinical prostate cancer in the testosterone group compared with the placebo groups • But no sufficient data to drawn conclusion
  • 112. Ejaculatory disorder • Hematospermia • Retrograde ejaculation • Premature ejaculation • Ejaculatory failure
  • 113. Hematospermia • Commonly seen after prolonged period of sexual abstience, always resolved spontaneously • Investigation if beyond several weeks • History: – Exclude hematuria – Recent trauma – Infection (STD) – Bleeding disorder – PMH: TB, Ca prostate • Three important point need evaluation: – Patient age – Duration & recurrence – Associated hematuria
  • 114. • Physical examination: – Blood pressure – Genital: TB, bead cord vas – Penis – DRE: prostate • Investigation: – Bld, PSA – MSU : sterile pyuria – Urethral swab for younger pt – Cytology – FR + RU if slow stream
  • 115. Cause • Infection (40%): TB, HIV, CMV • STD: herpex, chlamydia, ureaplasma • Prostatitis (30%) • Post-TRUS + bx • Prostate cancer • Urethritis and urethral stricture • Acquire or congenital cyst of the seminal vesicle • Systemic disorder: HT, liver disease, lymphoma
  • 116. Further investigation • TRUS: reveal abnormalities in 95% – Prostatic calcification (40%) – Ejaculatory duct calculi (40%) – Dilated ejaculatory duct (30%) – Ejaculatory duct cyst (10%) – BPH (30%) – Dilated or clacified SV (20%) – Mullerian duct remnants (7%) • Treatment: – Reassurance – Antibiotics ?
  • 117. Retrograde ejaculation • History: – Low ejaculate volume – Post- ejaculate urine cloudy • Investigation : post-ejaculate urine examination for sperm • Cause : – Post- RPLND – DM – Bladder neck surgery, TURP – Trauma – Alpha- blockers – Urethral stricture – Spinal cord injury
  • 118. Treatment • Medication: to close the bladder neck – Sympathomimetic : pseudoephedrine & ephedrine – TCA: imipramine – Efficacy : 50% • IVF: – Sperm retrived from alkalinised post-ejaculate urine – Fertilization rate: 50% • How to alkalinise urine? – Sodium bicarbonate: 1gm at night before & 1mg in the morning of sperm collection – Or use Liverpool solution : NaCl + NaHCO3 – Empty bladder before masturbation – Obstain post-ejaculated urine & send to lab ASAP
  • 120. Premature Ejaculation • Most important point: 1. Short ejaculatory latency time 2. Lack of ejaculatory control 3. Decreased satisfaction with sexual intercourse • Interpersonal distress • Negative man’s self-esteem • Reduced sexual function and QOL Patrick DL et al ,J Sex med 2005 Giuliano F et al, Eur Urol 2008 Rowland DL et al, J Urol 2007
  • 121. DSM-IV-TR 2000 • Persistent or recurrent ejaculation with minimal sexual stimulation; –Before, on , or shortly after penetration –Before the person wishes it; • Must also cause marked distress or interpersonal difficulty; • Cannot be due exclusively to the direct effects of a substance.
  • 122. The International Society for Sexual Medicine (ISSM) • The first evidence-based definition • ‘Premature ejaculation is a male sexual dysfunction characterized by ejaculation which always or nearly always occurs prior to or within about one minute of vaginal penetration; and inability to delay ejaculation on all or nearly all vaginal penetrations; and negative personal consequences, such as distress, bother, frustration and/or the avoidance of sexual intimacy
  • 123. Classification • Lifelong condition • Acquired condition • Natural variable PE • Premature –like ejaculatory dysfunction Cooper AJ et al, J Sex Maritla Ther 1993 Waldinger MD et al, Drugs 2007
  • 124. What is the prevalence? • Major problem in assessing the prevalence of PE is the lack of an accurate (validated) definition • The most common male sexual dysfunction, with prevalence rates of 20- 30% • Prevalence of PE is not affected by age
  • 125. Etiology Psychogenic: • Anxiety • Early sexual experience • Infrequent sexual intercourse • Poor ejaculatory control technique • Negative conditioning Biological cause: • Penile hypersensitivity • Hyperexcitable ejaculatory reflx • Endocrinopathy • Genetic predisposition • 5HT- receptor dysfunction
  • 127. Neurophysiology • Ejaculatory control centers in spinal cord • Received peripheral afferents and supraspinal influences • Coordinate sympathetic , parasympathetic and somatic outputs to pelviperineal structrues
  • 128. 5-HT neurons • Activation of 5-HT1A autoreceptors  decrease 5-HT release by presynaptic neurons (-ve feedback) • Activation of 5-HT1A decrease ejaculatory latency • Activation of postsynaptic 5-HT2C or 5-HT1B receptors prolongs ejaculatory latency • PE may be due to imbalance btw 5-HT1A (hypersensitivity) and 5-HT2C or 5-HT1B (hyposensitivity) • Increase central 5-HT  delay ejaculation
  • 129. What is the approach to PE? • History and physical examination – Intravaginal ejaculatory latency time (IELT) • Clinical use of self-estimated IELT is adequate, stopwatch- measured IELT is necessary in clinical trials – Patient-reported outcomes (PROs) have the potential to identify men with PE • Patient-reported outcomes (PROs) have the potential to identify men with PE • Further research is needed before PROs can be recommended for clinical use – Duration time of ejaculation, degree of sexual stimulus, impact on sexual activity and QoL, and drug use or abuse – It is also important to distinguish PE from ED – ED develop secondary PE caused by the anxiety – Examination of the vascular, endocrine and neurological systems • Routine laboratory or neurophysiological tests are not recommended
  • 130. Measurement of response • Intravaginal ejaculatory latency time (IELT) – Time between vaginal intromission & ejaculation • Perceived controlled over ejaculation • Improvement of personal distress
  • 131. What are behavioural therapy? • ‘Stop-start’ programme developed by Semans • ‘Squeeze’ technique, proposed by Masters and Johnson • Masturbation before anticipation of sexual intercourse • Success rates of 50-60% in short term • Time intensive, require the support of a partner and can be difficult to do • Recurrence is likely after treatment cessation
  • 132. Pharmacotherapy for PE • Anti-depressant: –TCA: Clomipramine –SSRIs: paroxetine, fluoxetine, sertraline, etc • Phosphodiesterase-5 inhibitors (PDE-5i) • Tramadol • Topical agents: lidocaine/prilocaine
  • 133. SSRI • Increase synaptic 5-HT concentration via blockade of 5-HT transporters • Paroxetine (20-40mg), Clomipramine (10-50mg) or fluoxetine (20- 40mg) • Meta-analysis: Paroxetine produce strongest delay in ejaculation • Daily txn, effect start on 2 week • Need to withdrawn gradually over 4 week (except fluoxetine) • SE: – Psychiatric and neurological – Dermatological reaction – Anticholinergic SE – Change in body weight – Cognitive impairment – Drug-drug interactions – Sexual SE: ED and loss of libido
  • 134. SSRI discontinuation syndrome • Especially in paroxetine • 1-3 days after drug discontinuation • Median duration: > 1 week • Reversible when SSRI reintroduced • Dizziness, nausea and emesis, headache, gait instability, lethargy, agitation , anxiety and insomnia Black K et al, J Psy Neurosci 2000 Haddad P et al, J psychopahrmacol 1998 Tamam L et al, Adv ther 2002
  • 135. Serotonin syndrome • SSRI with long half-lives • Interactions with agents that enhance 5- HT CNS activity • Myoclonus, hyper-reflexia, sweating, shivering , lack of coordination and mental status changes Nelson EB et al, J Clin Psychiatry 1997 Lane R et al, J Clin Psychopharmacol 1997
  • 136. Dapoxetine • New agents under development • Rapid onset (1.29hr) and short half-live (1.49hr) • On-demand dapoextine 30 or 60mg significantly improved outcome vs placebo • IELT increase 3.6x from baseline • SE: nausea, diarrhoea, headache, dizziness and insomnia Pryor JL et al, Lancet 2006
  • 137. PDE-5i • Results has been conflicting • No pharmacological rationale • Paroxetine + sildenafil vs paroxetine: increased IELT and satisfaction but with more SE (headaches and flushing) Salonia A et al, J Urol 2002 • No effect in men without coexiting ED, cause decrease in post-ejaculatory refractory period Chen J et al, Urology 2002
  • 138. Tramadol • Centrally acting synthetic opioid • Inhibit nor-adrenaline and serotonin reuptake • Rapidly absorbed and eliminated • Increased IELT, sexual satisfaction and ejaculatory control vs placebo (p<0.05) Safarinejad MR et al, J Clin Psychopharmacol 2006 Salem EA et al, J Sex Med 2008
  • 139. Topical agents • Topical lidocaine/prilocaine cause desensitization • Increase mean IELT by 2.4x vs placebo (p<0.01) Dinsmore WW et al, BJU Int 2006 • SE: local numbness(12%), loss of erection • Severance Secret cream: increase IELT and sexual satisfaction vs placebo Choi HK et al, Urology 2000 Choi HK et al, Int J impot Res 1999
  • 140. Conclusion • PE is an under-treated condition due to lack of understanding of its cause and potential therapy, and because of its sensitive nature • 5-HT has been implicated as a key mediator of ejaculatory control
  • 141. Conclusion • Available therapy include off-label use of SSRIa and PDE-5i , as well as topical anaesthetics • New on-demand agents like tramadol and dapoxetine are currently under evaluation • Role of other central neurotransmitter as future targets to delay ejaculation needs further investigation
  • 142. Ejaculatory failure • Cause: Post SCI, RPLND , psychogenic • History: what level , bowel and bladder fxn • Investigation: – SA: azzospermia – Post-orgasmic urine  no fructose • Treatment: Electro-ejaculator – Seager electro-ejaculator – Rectal probe to stimulate perirectal, periprostatic sympathetic nerves – May require GA – Watch out for autonomic dysreflexia in above T6 lesion – Sperm: poorere quality & mobility • Alternative: sperm retrieval technique (pregnancy rate 70%)
  • 144. What is Peyronie’s disease? • Fibrous plaque within tunica albuginea of penis • associated with DM, antiepileptic drugs and beta blockers • Curvature, penile pain or shortening • Erectile dysfunction
  • 145. What is the cause? • Trauma to tunica albuginea • Wound healing > excessive fibrotic plaque • Dorsal plaque more common • Penile curvature as corpus cavernosum can’t lengthen fully on erection limited by plaque • Associated with Dupuytren’s contracture 30% • Incidence <5%, men aged 40-70 years
  • 146. What is natural course of the disease? • Active phase 6 months, painful erection with changing deformity • Quiescent phase 9-12 months stable deformity, painless • Natural Hx over 18 m – 13% improved – 40% stable – 47% progress
  • 147. How to make diagnosis? • By history and P/E – History • Disease Duration • Pain • Penile deformity –angle, direction • Stability • Penile length • Erections , able to penetrate • IIEF • Risk factor for ED – P/E • Assess degree of curvature by 1 photogragh, 2 IC PGE1 • Exam for plaques, location, size • Penile length, stretched & flaccid state • Extremeties for Dupuytren’s contracture
  • 148. What is the treatment? • Early disease <3m consider medical Tx / injection / ESWL, low successful rate – Oral vit E • 200mg tds for 3 months • In a randomized trial vitamin E has been shown to improve pain in 75% of patients and improves the deformity in 10% – Oral colchicine x3m (limited evidence for efficacy) – Intralesional verapamil, steroid x 6m
  • 149. Photograph taken during a procedure • What is being done? (1) • What procedure is this? (0.5) • Name 3 complications from this procedure (0.5 each) Q45
  • 150. • Artificial erection from injection of saline into corpus cavernosum (1) • Correction of penile curvature eg. in Peyronie’s disease (0.5) • Shortening of penis, erectile dysfunction, deformity recurrence, palpable suture through penile shaft skin, altered/decreased penile sensation (0.5 each, total 1.5)
  • 151. Is ESWL useful? • Initiating an inflammatory reaction thru direct damage to plaque and result in plaque resorption • No study has demostrate any improvement in plaque size or curvature • NICE do not recommend
  • 152. What are the indications for Surgery? • Disease present for at least 12m • stable for at least 3m • Deformity makes intercourse difficult • Quality of erection important • ED > ? Prosthesis
  • 153. Surgery: Penile shortening • Indicated in pt: no ED , <60degree curvature, no hourglass demormities or hinge effect • Must warn pt of Penile shortening effect • Nesbit – Penis degloved via circumglandular incision – Artificial erection with NS – ellipitcal incision: 1mm for 10 degree deformity curvature on convex side – Complication: all penile shortening, 1% ED rate, recurrence of deformity – Success rate 80%
  • 154. Surgery: Penile maintaining • Lue’s procedure – By incising the plaque and interposing a graft (fascia lata / vein graft/ Gortex graft) – Do cause penile shortening but not to the extent that corporal plication – More ED – 15% – Not recommended for complete excision of the plaque due to compromising veno-occlusive mechanism and causing ED • Surgery in general – Success rate: 80% – Risk: bleeding, infection , bruising – Loss of 1cm in 26% – ED in 15% – Recommend penile traction device or penile rehab with PDE5-i • About 10% of patients will subsequently require circumcision due to secondary phimosis
  • 155. When is prothesis required? • Penile prosthesis is indicated in patients with both Peyronie’s disease and severe ED • After insertion and inflation of penile prosthesis, the penis is bent in opposite direction to break the plaque (modelling) – 90% successful rate
  • 157. Priapism • Definition: – Persistent erection > 4 hours – Not related to sexual desire • Two age group: – 5-10 yo – 20-50 yo 3. Stuttering priaprism – repeated, shorter self-
  • 158. Presentation • FOUR Main questions: 1. Duration of erection > 4 hours? 2. Painful / non painful ? (Ischemic vs nonischemic) 3. Previous history of priapism 4. Predisposing factors • Physical examination: – Rigid corpora cavernosa – Flaccid Corpus spongiosum and glans penis
  • 159.
  • 160. Investigation • Blood: CBP, Hb/electrophoresis (SSD) • Urine: C/ST , toxicology • Penile blood gas: – Aspirate blood directly from either corpora • Duplex USG of carvernosal arteries: – Ischemic ( inflow low or absent) – Non ischemic (inflow normal or high) • Penile pudenal arteriography: not readily available Appearance pH PO2 (mmHg) PCO2 (mmHg) Low flow Dark red <7.25 <30 >60 High flow Bright red = 7.4 >90 <40
  • 161. Treatment • Conservative • Medical • Minimally invasive • Surgical treatment • Always warn patient about the possibility of impotence due to cavernosal fibrosis
  • 162. Doppler USG can differentiate high or low flow priaprism
  • 163.
  • 164. Treatment for Priapism • Cavernosal aspiration successful rate: 1/3 • Distal shunt – Winter (large biopsy needle , corporo-glanular) – Ebbehoj (Scalpel, corporo-glanular) • Lue’s modification “T-Shunt” (scalpel, corporo-glandular) – El-Ghorab: piece of tunic albuginea excised at tips of coprora via a dorsal transverse incision just distal to corona • Proximal shunt: – For failed distal shunt or severe distal penile edema – 80% successful rate, but ED > 90% – Quackels /Sacher (corporo-spongiosal) – Grayhack (corporo-saphenous) • Supra-selective embolisation of common penile artery – successful rate 80% – Absorbable materials like clots and gel cause less ED than coils or permanent chemical
  • 165.
  • 169. A procedure for a urologic emergency is about to be performed • What is the name of this procedure? (2) • What is it used for? (1) Q20
  • 170. • Ebbehoj shunt, a type of distal cavernoglandular shunt (2, 1 mark for mentioning just “shunt” without name) • Ischemic priapism not responsive to injectional medical treatment (1)
  • 171. How about high-flow priaprism? • Not a urological emergency • Duplex USG can confirm diagnosis • Selective internal pudendal embolisation, better with absorbable material including clots or gels • Successful rate up to 80% • If failed > open exploration and direct ligation

Hinweis der Redaktion

  1. Key Presentation Points Cavernosal tissue is sponge-like, with a mesh of interconnected cavernosal spaces, which are also called cavernous sinuses or lacunar spaces. The cavernosal spaces are lined with vascular epithelial cells and are separated by trabeculae, which are composed of bundles of smooth muscle fibers, with an extracellular matrix of elastin, collagen, and fibroblasts. Blood supply to the penis is provided by the cavernosal arteries, which are branches of the penile artery that itself originates from the internal pudendal artery. The cavernosal arteries run the length of the corpora cavernosa and subsequently divide into twisting branches called helicine arteries. Three sets of veins drain blood from the penis—the deep, intermediate, and superficial veins. The deep veins drain the corpora cavernosa and the corpus spongiosum. Andersson K-E, Wagner G. Physiology of penile erection. Physiol Rev 1995;75:191-236.
  2. The most common factor involved in organic erectile dysfunction (ED) is impaired blood flow into the penis, which is common in patients with atherosclerosis (associated with approximately 40% of ED cases in men over 50 years of age1) diabetes (occurring in an estimated 50% [range, 28% - 59%] of patients with diabetes, regardless of type, with prevalence dependent on age and diabetes severity2-5) Other chronic illnesses and medical conditions associated with ED include: chronic renal failure (40%)6 multiple sclerosis (71%)8 chronic obstructive respiratory disease (30%)10 penile abnormalities, such as those found in Peyronie’s disease11 endocrine disorders (eg, hypogonadism, hyperprolactinemia, hypothyroidism, and hyperthyroidism) psychiatric disorders (~ 90% of men with severe depression).12
  3. .
  4. Both men and his partners
  5. Lifelong condition : present since the onset of sexual maturity Acquired condition : that develops after an interval of normal sexual function Natural variable PE: occurs in specific situations. Premature-like ED: men with ejaculatory latency times in the normal range perceive their ejaculation to be premature
  6. Physiology of ejaculation in unimpaired in patient with ED, however the lack of voluntary control is distressing Shamloul R: Chronic prostatitis in PE : a cohort study in 153 men
  7. The ejaculatory center in the spinal cord (L2 –L3) is activated when arousal reaches a certain threshold (!A2). Immediately prior to ejaculation, efferent sympathetic impulses trigger the partial evacuation of the prostate gland and the emission of semen from the vas deferens to the posterior part of the urethra. This triggers the ejaculation reflex and is accompanied by orgasm, the apex of sexual excitement. The effects of orgasm can be felt throughout the entire body, which is reflected by perspiration and an increase in respiratory rate, heart rate, blood pressure, and skeletal muscle tone. During ejaculation, the internal sphincter muscle closes off the urinary bladder while the vas deferens, seminal vesicles and bulbocavernous and ischiocavernous muscles contract rhythmically to propel the semen out of the urethra.
  8. 5-HT1A &amp; 5-HT2C plays a key role in regulating ejaculation at the central level
  9. 12-week placebo-controlled trails in USA of 2614 men
  10. Winter Shunt El-Ghorab Quackels /Sacher Grayhack